Programs & Examples On #Border layout

BorderLayout is a Java Swing/AWT layout manager that lays out a container, arranging and resizing its components to fit in five regions: north, south, east, west, and center.

How to uninstall / completely remove Oracle 11g (client)?

Assuming a Windows installation, do please refer to this:

http://www.oracle-base.com/articles/misc/ManualOracleUninstall.php

  • Uninstall all Oracle components using the Oracle Universal Installer (OUI).
  • Run regedit.exe and delete the HKEY_LOCAL_MACHINE\SOFTWARE\ORACLE key. This contains registry entires for all Oracle products.
  • Delete any references to Oracle services left behind in the following part of the registry: HKEY_LOCAL_MACHINE\SYSTEM\CurrentControlSet\Services\Ora* It should be pretty obvious which ones relate to Oracle.
  • Reboot your machine.
  • Delete the "C:\Oracle" directory, or whatever directory is your ORACLE_BASE.
  • Delete the "C:\Program Files\Oracle" directory.
  • Empty the contents of your "C:\temp" directory.
  • Empty your recycle bin.

Calling additional attention to some great comments that were left here:

  • Be careful when following anything listed here (above or below), as doing so may remove or damage any other Oracle-installed products.
  • For 64-bit Windows (x64), you need also to delete the HKEY_LOCAL_MACHINE\SOFTWARE\Wow6432Node\ORACLE key from the registry.
  • Clean-up by removing any related shortcuts that were installed to the Start Menu.
  • Clean-up environment variables:
    • Consider removing %ORACLE_HOME%.
    • Remove any paths no longer needed from %PATH%.

This set of instructions happens to match an almost identical process that I had reverse-engineered myself over the years after a few messed-up Oracle installs, and has almost always met the need.

Note that even if the OUI is no longer available or doesn't work, simply following the remaining steps should still be sufficient.

(Revision #7 reverted as to not misquote the original source, and to not remove credit to the other comments that contributed to the answer. Further edits are appreciated (and then please remove this comment), if a way can be found to maintain these considerations.)

How to delete/unset the properties of a javascript object?

To blank it:

myObject["myVar"]=null;

To remove it:

delete myObject["myVar"]

as you can see in duplicate answers

.do extension in web pages?

.do comes from the Struts framework. See this question: Why do Java webapps use .do extension? Where did it come from? Also you can change what your urls look like using mod_rewrite (on Apache).

How to print the values of slices

I prefer fmt.Printf("%+q", arr) which will print

["some" "values" "list"]

https://play.golang.org/p/XHfkENNQAKb

Difference between map and collect in Ruby?

#collect is actually an alias for #map. That means the two methods can be used interchangeably, and effect the same behavior.

How to add a new line in textarea element?

Break enter Keyword line in Textarea using CSS:

white-space: pre-wrap;

How do I escape a single quote in SQL Server?

2 ways to work around this:


for ' you can simply double it in the string, e.g. select 'I''m happpy' -- will get: I'm happy


For any charactor you are not sure of: in sql server you can get any char's unicode by select unicode(':') (you keep the number)

So this case you can also select 'I'+nchar(39)+'m happpy'

Generate a UUID on iOS from Swift

Also you can use it lowercase under below

let uuid = NSUUID().UUIDString.lowercaseString
print(uuid)

Output

68b696d7-320b-4402-a412-d9cee10fc6a3

Thank you !

How to change Status Bar text color in iOS

Just to summarize, edit your project Info.plist and add:

View controller-based status bar appearance : NO

Status bar style : Opaque black style

or if you have raw key/value plist

UIViewControllerBasedStatusBarAppearance : NO

UIStatusBarStyle : Opaque black style

Program to find prime numbers

You only need to check odd divisors up to the square root of the number. In other words your inner loop needs to start:

for (int j = 3; j <= Math.Sqrt(i); j+=2) { ... }

You can also break out of the function as soon as you find the number is not prime, you don't need to check any more divisors (I see you're already doing that!).

This will only work if num is bigger than two.

No Sqrt

You can avoid the Sqrt altogether by keeping a running sum. For example:

int square_sum=1;
for (int j=3; square_sum<i; square_sum+=4*(j++-1)) {...}

This is because the sum of numbers 1+(3+5)+(7+9) will give you a sequence of odd squares (1,9,25 etc). And hence j represents the square root of square_sum. As long as square_sum is less than i then j is less than the square root.

Get a list of dates between two dates using a function

SELECT  dateadd(dd,DAYS,'2013-09-07 00:00:00') DATES
INTO        #TEMP1
FROM
(SELECT TOP 365 colorder - 1 AS DAYS from master..syscolumns 
    WHERE id = -519536829 order by colorder) a

WHERE datediff(dd,dateadd(dd,DAYS,'2013-09-07 00:00:00'),'2013-09-13 00:00:00' ) >= 0 
    AND  dateadd(dd,DAYS,'2013-09-07 00:00:00') <=  '2013-09-13 00:00:00'  
    SELECT * FROM #TEMP1

Can't run Curl command inside my Docker Container

Ran into this same issue while using the CURL command inside my Dockerfile. As Gilles pointed out, we have to install curl first. These are the commands to be added in the 'Dockerfile'.

FROM ubuntu:16.04

# Install prerequisites
RUN apt-get update && apt-get install -y \
curl
CMD /bin/bash

www-data permissions?

As stated in an article by Slicehost:

User setup

So let's start by adding the main user to the Apache user group:

sudo usermod -a -G www-data demo

That adds the user 'demo' to the 'www-data' group. Do ensure you use both the -a and the -G options with the usermod command shown above.

You will need to log out and log back in again to enable the group change.

Check the groups now:

groups
...
# demo www-data

So now I am a member of two groups: My own (demo) and the Apache group (www-data).

Folder setup

Now we need to ensure the public_html folder is owned by the main user (demo) and is part of the Apache group (www-data).

Let's set that up:

sudo chgrp -R www-data /home/demo/public_html

As we are talking about permissions I'll add a quick note regarding the sudo command: It's a good habit to use absolute paths (/home/demo/public_html) as shown above rather than relative paths (~/public_html). It ensures sudo is being used in the correct location.

If you have a public_html folder with symlinks in place then be careful with that command as it will follow the symlinks. In those cases of a working public_html folder, change each folder by hand.

Setgid

Good so far, but remember the command we just gave only affects existing folders. What about anything new?

We can set the ownership so anything new is also in the 'www-data' group.

The first command will change the permissions for the public_html directory to include the "setgid" bit:

sudo chmod 2750 /home/demo/public_html

That will ensure that any new files are given the group 'www-data'. If you have subdirectories, you'll want to run that command for each subdirectory (this type of permission doesn't work with '-R'). Fortunately new subdirectories will be created with the 'setgid' bit set automatically.

If we need to allow write access to Apache, to an uploads directory for example, then set the permissions for that directory like so:

sudo chmod 2770 /home/demo/public_html/domain1.com/public/uploads

The permissions only need to be set once as new files will automatically be assigned the correct ownership.

What's the best CRLF (carriage return, line feed) handling strategy with Git?

Don't convert line endings. It's not the VCS's job to interpret data -- just store and version it. Every modern text editor can read both kinds of line endings anyway.

Iterating over arrays in Python 3

When you loop in an array like you did, your for variable(in this example i) is current element of your array.

For example if your ar is [1,5,10], the i value in each iteration is 1, 5, and 10. And because your array length is 3, the maximum index you can use is 2. so when i = 5 you get IndexError. You should change your code into something like this:

for i in ar:
    theSum = theSum + i

Or if you want to use indexes, you should create a range from 0 ro array length - 1.

for i in range(len(ar)):
    theSum = theSum + ar[i]

Exit while loop by user hitting ENTER key

This works for python 3.5 using parallel threading. You could easily adapt this to be sensitive to only a specific keystroke.

import time
import threading


# set global variable flag
flag = 1

def normal():
    global flag
    while flag==1:
        print('normal stuff')
        time.sleep(2)
        if flag==False:
            print('The while loop is now closing')

def get_input():
    global flag
    keystrk=input('Press a key \n')
    # thread doesn't continue until key is pressed
    print('You pressed: ', keystrk)
    flag=False
    print('flag is now:', flag)

n=threading.Thread(target=normal)
i=threading.Thread(target=get_input)
n.start()
i.start()

Monitor network activity in Android Phones

Preconditions: adb and wireshark are installed on your computer and you have a rooted android device.

  1. Download tcpdump to ~/Downloads
  2. adb push ~/Downloads/tcpdump /sdcard/
  3. adb shell
  4. su root
  5. mv /sdcard/tcpdump /data/local/
  6. cd /data/local/
  7. chmod +x tcpdump
  8. ./tcpdump -vv -i any -s 0 -w /sdcard/dump.pcap
  9. Ctrl+C once you've captured enough data.
  10. exit
  11. exit
  12. adb pull /sdcard/dump.pcap ~/Downloads/

Now you can open the pcap file using Wireshark.

As for your question about monitoring specific processes, find the bundle id of your app, let's call it com.android.myapp

  1. ps | grep com.android.myapp
  2. copy the first number you see from the output. Let's call it 1234. If you see no output, you need to start the app.
  3. Download strace to ~/Downloads and put into /data/local using the same way you did for tcpdump above.
  4. cd /data/local
  5. ./strace -p 1234 -f -e trace=network -o /sdcard/strace.txt

Now you can look at strace.txt for ip addresses, and filter your wireshark log for those IPs.

How to copy a string of std::string type in C++?

strcpy example:

#include <stdio.h>
#include <string.h>

int main ()
{
  char str1[]="Sample string" ;
  char str2[40] ;
  strcpy (str2,str1) ;
  printf ("str1: %s\n",str1) ;
  return 0 ;
}

Output: str1: Sample string

Your case:

A simple = operator should do the job.

string str1="Sample string" ;
string str2 = str1 ;

How do you replace all the occurrences of a certain character in a string?

You really should have multiple input, e.g. one for firstname, middle names, lastname and another one for age. If you want to have some fun though you could try:

>>> input_given="join smith 25"
>>> chars="".join([i for i in input_given if not i.isdigit()])
>>> age=input_given.translate(None,chars)
>>> age
'25'
>>> name=input_given.replace(age,"").strip()
>>> name
'join smith'

This would of course fail if there is multiple numbers in the input. a quick check would be:

assert(age in input_given)

and also:

assert(len(name)<len(input_given))

Replace multiple strings at once

For the tags, you should be able to just set the content with .text() instead of .html().

Example: http://jsfiddle.net/Phf4u/1/

var textarea = $('textarea').val().replace(/<br\s?\/?>/, '\n');

$("#output").text(textarea);

...or if you just wanted to remove the <br> elements, you could get rid of the .replace(), and temporarily make them DOM elements.

Example: http://jsfiddle.net/Phf4u/2/

var textarea = $('textarea').val();

textarea = $('<div>').html(textarea).find('br').remove().end().html();

$("#output").text(textarea);

Failed to connect to camera service

since this question was asked 4 years back..and i didn't realised that unless mentioned by the Questioner..when there were no Run time permissions support.

but hoping it useful for the users who still caught in this situation.. Have a look at Run Time Permissions ,for me it solved the problem when i added Run time permissions to grant camera access. Alternatively you can grant permissions to the app manually by going to your mobile settings=>Apps=>(select your app)=>Permissions section in the appeared window and enable/disable desired permissions. hope this will work.

Count characters in textarea

$.fn.extend( {
       limiter: function(limit, elem) {
            $(this).on("keyup focus", function() {
               setCount(this, elem);
           });
            function setCount(src, elem) {
               var chars = src.value.length;
                if (chars > limit) {
                    src.value = src.value.substr(0, limit);
                    chars = limit;
                }
                elem.html( limit - chars );
            }
            setCount($(this)[0], elem);
        }
    });

    var elem = $("#cntr");  
    $("#status_txt").limiter(160, elem);

How can I set the request header for curl?

Just use the -H parameter several times:

curl -H "Accept-Charset: utf-8" -H "Content-Type: application/x-www-form-urlencoded" http://www.some-domain.com

How to call an async method from a getter or setter?

When I ran into this problem, trying to run an async method synchronicity from either a setter or a constructor got me into a deadlock on the UI thread, and using an event handler required too many changes in the general design.
The solution was, as often is, to just write explicitly what I wanted to happen implicitly, which was to have another thread handle the operation and to get the main thread to wait for it to finish:

string someValue=null;
var t = new Thread(() =>someValue = SomeAsyncMethod().Result);
t.Start();
t.Join();

You could argue that I abuse the framework, but it works.

Git status ignore line endings / identical files / windows & linux environment / dropbox / mled

Use .gitattributes instead, with the following setting:

# Ignore all differences in line endings
*        -crlf

.gitattributes would be found in the same directory as your global .gitconfig. If .gitattributes doesn't exist, add it to that directory. After adding/changing .gitattributes you will have to do a hard reset of the repository in order to successfully apply the changes to existing files.

Difference between natural join and inner join

difference is that int the inner(equi/default)join and natural join that in the natuarl join common column win will be display in single time but inner/equi/default/simple join the common column will be display double time.

Doctrine 2: Update query with query builder

With a small change, it worked fine for me

$qb=$this->dm->createQueryBuilder('AppBundle:CSSDInstrument')
               ->update()
               ->field('status')->set($status)
               ->field('id')->equals($instrumentId)
               ->getQuery()
               ->execute();

jQuery: selecting each td in a tr

Your $(magicSelector) could be $('td', this). This will grab all td that are children of this, which in your case is each tr. This is the same as doing $(this).find('td').

$('td', this).each(function() {
// Logic
});

Import data.sql MySQL Docker Container

you can follow these simple steps:

FIRST WAY :

first copy the SQL dump file from your local directory to the mysql container. use docker cp command

docker cp [SRC-Local path to sql file] [container-name or container-id]:[DEST-path to copy to]

docker cp ./data.sql mysql-container:/home

and then execute the mysql-container using (NOTE: in case you are using alpine version you need to replace bash with sh in the given below command.)

docker exec -it -u root mysql-container bash

and then you can simply import this SQL dump file.

mysql [DB_NAME] < [SQL dump file path]

mysql movie_db < /home/data.sql

SECOND WAY : SIMPLE

docker cp ./data.sql mysql-container:/docker-entrypoint-initdb.d/

As mentioned in the mysql Docker hub official page.

Whenever a container starts for the first time, a new database is created with the specified name in MYSQL_DATABASE variable - which you can pass by setting up the environment variable see here how to set environment variables

By default container will execute files with extensions .sh, .sql and .sql.gz that are found in /docker-entrypoint-initdb.d folder. Files will be executed in alphabetical order. this way your SQL files will be imported by default to the database specified by the MYSQL_DATABASE variable.

for more details you can always visit the official page

Fluid or fixed grid system, in responsive design, based on Twitter Bootstrap

When you decide between fixed width and fluid width you need to think in terms of your ENTIRE page. Generally, you want to pick one or the other, but not both. The examples you listed in your question are, in-fact, in the same fixed-width page. In other words, the Scaffolding page is using a fixed-width layout. The fixed grid and fluid grid on the Scaffolding page are not meant to be examples, but rather the documentation for implementing fixed and fluid width layouts.

The proper fixed width example is here. The proper fluid width example is here.

When observing the fixed width example, you should not see the content changing sizes when your browser is greater than 960px wide. This is the maximum (fixed) width of the page. Media queries in a fixed-width design will designate the minimum widths for particular styles. You will see this in action when you shrink your browser window and see the layout snap to a different size.

Conversely, the fluid-width layout will always stretch to fit your browser window, no matter how wide it gets. The media queries indicate when the styles change, but the width of containers are always a percentage of your browser window (rather than a fixed number of pixels).

The 'responsive' media queries are all ready to go. You just need to decide if you want to use a fixed width or fluid width layout for your page.

Previously, in bootstrap 2, you had to use row-fluid inside a fluid container and row inside a fixed container. With the introduction of bootstrap 3, row-fluid was removed, do no longer use it.

EDIT: As per the comments, some jsFiddles for:

These fiddles are completely Bootstrap-free, based on pure CSS media queries, which makes them a good starting point, for anyone willing to craft similar solution without using Twitter Bootstrap.

Javascript: Extend a Function

With a wider view of what you're actually trying to do and the context in which you're doing it, I'm sure we could give you a better answer than the literal answer to your question.

But here's a literal answer:

If you're assigning these functions to some property somewhere, you can wrap the original function and put your replacement on the property instead:

// Original code in main.js
var theProperty = init;

function init(){
     doSomething();
}

// Extending it by replacing and wrapping, in extended.js
theProperty = (function(old) {
    function extendsInit() {
        old();
        doSomething();
    }

    return extendsInit;
})(theProperty);

If your functions aren't already on an object, you'd probably want to put them there to facilitate the above. For instance:

// In main.js
var MyLibrary = {
    init: function init() {
    }
};

// In extended.js
(function() {
    var oldInit = MyLibrary.init;
    MyLibrary.init = extendedInit;
    function extendedInit() {
        oldInit.call(MyLibrary); // Use #call in case `init` uses `this`
        doSomething();
    }
})();

But there are better ways to do that. Like for instance, providing a means of registering init functions.

// In main.js
var MyLibrary = (function() {
    var initFunctions = [];
    return {
        init: function init() {
            var fns = initFunctions;
            initFunctions = undefined;
            for (var index = 0; index < fns.length; ++index) {
                try { fns[index](); } catch (e) { }
            }
        },
        addInitFunction: function addInitFunction(fn) {
            if (initFunctions) {
                // Init hasn't run yet, remember it
                initFunctions.push(fn);
            } else {
                // `init` has already run, call it almost immediately
                // but *asynchronously* (so the caller never sees the
                // call synchronously)
                setTimeout(fn, 0);
            }
        }
    };
})();

Here in 2020 (or really any time after ~2016), that can be written a bit more compactly:

// In main.js
const MyLibrary = (() => {
    let initFunctions = [];
    return {
        init() {
            const fns = initFunctions;
            initFunctions = undefined;
            for (const fn of fns) {
                try { fn(); } catch (e) { }
            }
        },
        addInitFunction(fn) {
            if (initFunctions) {
                // Init hasn't run yet, remember it
                initFunctions.push(fn);
            } else {
                // `init` has already run, call it almost immediately
                // but *asynchronously* (so the caller never sees the
                // call synchronously)
                setTimeout(fn, 0);
                // Or: `Promise.resolve().then(() => fn());`
                // (Not `.then(fn)` just to avoid passing it an argument)
            }
        }
    };
})();

Reliable way to convert a file to a byte[]

looks good enough as a generic version. You can modify it to meet your needs, if they're specific enough.

also test for exceptions and error conditions, such as file doesn't exist or can't be read, etc.

you can also do the following to save some space:

 byte[] bytes = System.IO.File.ReadAllBytes(filename);

'import' and 'export' may only appear at the top level

This is not direct answer to the original question but I hope this suggestion helps someones with similar error:

When using a newer web-api with Webpack+Babel for transpiling and you get

Module parse failed: 'import' and 'export' may only appear at the top level

then you probably forgot to import a polyfill.

For example:
when using fetch() api and targeting for es2015, you should

  1. add whatwg-fetch polyfill to package.json
  2. add import {fetch} from 'whatwg-fetch'

Defining Z order of views of RelativeLayout in Android

If you want to do this in code you can do

View.bringToFront();

see docs

How to change a <select> value from JavaScript

$('#select').val('defaultValue'); 
$('#select').change();

This will also trigger events hooked to this select

Rename specific column(s) in pandas

data.rename(columns={'gdp':'log(gdp)'}, inplace=True)

The rename show that it accepts a dict as a param for columns so you just pass a dict with a single entry.

Also see related

How do I POST XML data to a webservice with Postman?

Send XML requests with the raw data type, then set the Content-Type to text/xml.


  1. After creating a request, use the dropdown to change the request type to POST.

    Set request type to POST

  2. Open the Body tab and check the data type for raw.

    Setting data type to raw

  3. Open the Content-Type selection box that appears to the right and select either XML (application/xml) or XML (text/xml)

    Selecting content-type text/xml

  4. Enter your raw XML data into the input field below

    Example of XML request in Postman

  5. Click Send to submit your XML Request to the specified server.

    Clicking the Send button

java: Class.isInstance vs Class.isAssignableFrom

clazz.isAssignableFrom(Foo.class) will be true whenever the class represented by the clazz object is a superclass or superinterface of Foo.

clazz.isInstance(obj) will be true whenever the object obj is an instance of the class clazz.

That is:

clazz.isAssignableFrom(obj.getClass()) == clazz.isInstance(obj)

is always true so long as clazz and obj are nonnull.

Why is January month 0 in Java Calendar?

In addition to DannySmurf's answer of laziness, I'll add that it's to encourage you to use the constants, such as Calendar.JANUARY.

How to get File Created Date and Modified Date

You could use below code:

DateTime creation = File.GetCreationTime(@"C:\test.txt");
DateTime modification = File.GetLastWriteTime(@"C:\test.txt");

Laravel 5 Failed opening required bootstrap/../vendor/autoload.php

I encountered the same problem. It occurred because composer was not able to install the dependencies specified in composer.json file. try running

composer install 

If this does not solve the problem, make sure the following php modules are installed php-mbstring php-dom

To install this extensions run the following in terminal

sudo apt-get install php-mbstring php-dom

once the installation is complete

try running the command in your project root folder

composer install 

shorthand If Statements: C#

Use the ternary operator

direction == 1 ? dosomething () : dosomethingelse ();

How to overplot a line on a scatter plot in python?

A one-line version of this excellent answer to plot the line of best fit is:

plt.plot(np.unique(x), np.poly1d(np.polyfit(x, y, 1))(np.unique(x)))

Using np.unique(x) instead of x handles the case where x isn't sorted or has duplicate values.

The call to poly1d is an alternative to writing out m*x + b like in this other excellent answer.

Find distance between two points on map using Google Map API V2

to get distance between two points try this code..

public static float GetDistanceFromCurrentPosition(double lat1,double lng1, double lat2, double lng2)
 {
        double earthRadius = 3958.75;

        double dLat = Math.toRadians(lat2 - lat1);

        double dLng = Math.toRadians(lng2 - lng1);

        double a = Math.sin(dLat / 2) * Math.sin(dLat / 2)
                + Math.cos(Math.toRadians(lat1))
                * Math.cos(Math.toRadians(lat2)) * Math.sin(dLng / 2)
                * Math.sin(dLng / 2);

        double c = 2 * Math.atan2(Math.sqrt(a), Math.sqrt(1 - a));

        double dist = earthRadius * c;

        int meterConversion = 1609;

        return new Float(dist * meterConversion).floatValue();

    }

Preloading images with JavaScript

CSS2 Alternative: http://www.thecssninja.com/css/even-better-image-preloading-with-css2

body:after {
  content: url(img01.jpg) url(img02.jpg) url(img03.jpg);
  display: none; 
}

CSS3 Alternative: https://perishablepress.com/preload-images-css3/ (H/T Linh Dam)

.preload-images {
  display: none; 
  width: 0;
  height: 0;
  background: url(img01.jpg),
              url(img02.jpg),
              url(img03.jpg);
}

NOTE: Images in a container with display:none might not preload. Perhaps visibility:hidden will work better but I have not tested this. Thanks Marco Del Valle for pointing this out

How do I make a C++ macro behave like a function?

Here is an answer coming right from the libc6! Taking a look at /usr/include/x86_64-linux-gnu/bits/byteswap.h, I found the trick you were looking for.

A few critics of previous solutions:

  • Kip's solution does not permit evaluating to an expression, which is in the end often needed.
  • coppro's solution does not permit assigning a variable as the expressions are separate, but can evaluate to an expression.
  • Steve Jessop's solution uses the C++11 auto keyword, that's fine, but feel free to use the known/expected type instead.

The trick is to use both the (expr,expr) construct and a {} scope:

#define MACRO(X,Y) \
  ( \
    { \
      register int __x = static_cast<int>(X), __y = static_cast<int>(Y); \
      std::cout << "1st arg is:" << __x << std::endl; \
      std::cout << "2nd arg is:" << __y << std::endl; \
      std::cout << "Sum is:" << (__x + __y) << std::endl; \
      __x + __y; \
    } \
  )

Note the use of the register keyword, it's only a hint to the compiler. The X and Y macro parameters are (already) surrounded in parenthesis and casted to an expected type. This solution works properly with pre- and post-increment as parameters are evaluated only once.

For the example purpose, even though not requested, I added the __x + __y; statement, which is the way to make the whole bloc to be evaluated as that precise expression.

It's safer to use void(); if you want to make sure the macro won't evaluate to an expression, thus being illegal where an rvalue is expected.

However, the solution is not ISO C++ compliant as will complain g++ -pedantic:

warning: ISO C++ forbids braced-groups within expressions [-pedantic]

In order to give some rest to g++, use (__extension__ OLD_WHOLE_MACRO_CONTENT_HERE) so that the new definition reads:

#define MACRO(X,Y) \
  (__extension__ ( \
    { \
      register int __x = static_cast<int>(X), __y = static_cast<int>(Y); \
      std::cout << "1st arg is:" << __x << std::endl; \
      std::cout << "2nd arg is:" << __y << std::endl; \
      std::cout << "Sum is:" << (__x + __y) << std::endl; \
      __x + __y; \
    } \
  ))

In order to improve my solution even a bit more, let's use the __typeof__ keyword, as seen in MIN and MAX in C:

#define MACRO(X,Y) \
  (__extension__ ( \
    { \
      __typeof__(X) __x = (X); \
      __typeof__(Y) __y = (Y); \
      std::cout << "1st arg is:" << __x << std::endl; \
      std::cout << "2nd arg is:" << __y << std::endl; \
      std::cout << "Sum is:" << (__x + __y) << std::endl; \
      __x + __y; \
    } \
  ))

Now the compiler will determine the appropriate type. This too is a gcc extension.

Note the removal of the register keyword, as it would the following warning when used with a class type:

warning: address requested for ‘__x’, which is declared ‘register’ [-Wextra]

How to change ProgressBar's progress indicator color in Android

I copied this from one of my apps, so there's prob a few extra attributes, but should give you the idea. This is from the layout that has the progress bar:

<ProgressBar
    android:id="@+id/ProgressBar"
    style="?android:attr/progressBarStyleHorizontal"
    android:layout_width="fill_parent"
    android:layout_height="wrap_content"
    android:indeterminate="false"
    android:maxHeight="10dip"
    android:minHeight="10dip"
    android:progress="50"
    android:progressDrawable="@drawable/greenprogress" />

Then create a new drawable with something similar to the following (In this case greenprogress.xml):

<?xml version="1.0" encoding="utf-8"?>
<layer-list xmlns:android="http://schemas.android.com/apk/res/android">
    <item android:id="@android:id/background">
        <shape>
            <corners android:radius="5dip" />
            <gradient
                android:angle="270"
                android:centerColor="#ff5a5d5a"
                android:centerY="0.75"
                android:endColor="#ff747674"
                android:startColor="#ff9d9e9d" />
        </shape>
    </item>

    <item android:id="@android:id/secondaryProgress">
        <clip>
            <shape>
                <corners android:radius="5dip" />
                <gradient
                    android:angle="270"
                    android:centerColor="#80ffb600"
                    android:centerY="0.75"
                    android:endColor="#a0ffcb00"
                    android:startColor="#80ffd300" />
            </shape>
        </clip>
    </item>
    <item android:id="@android:id/progress">
        <clip>
            <shape>
                <corners android:radius="5dip" />
                <gradient
                    android:angle="270"
                    android:endColor="#008000"
                    android:startColor="#33FF33" />
            </shape>
        </clip>
    </item>

</layer-list>

You can change up the colors as needed, this will give you a green progress bar.

MySQL 1062 - Duplicate entry '0' for key 'PRIMARY'

this step work perfectly for me.. you can try it

  1. Add indexing
  2. Add Autoincrement
  3. Add Primary Key

hope it work for you too. good luck

When to use IList and when to use List

You should use the interface only if you need it, e.g., if your list is casted to an IList implementation other than List. This is true when, for example, you use NHibernate, which casts ILists into an NHibernate bag object when retrieving data.

If List is the only implementation that you will ever use for a certain collection, feel free to declare it as a concrete List implementation.

Android Camera Preview Stretched

@Hesam 's answer is correct, CameraPreview will work in all portrait devices, but if the device is in landscape mode or in multi-window mode, this code is working perfect, just replace onMeasure()

@Override
protected void onMeasure(int widthMeasureSpec, int heightMeasureSpec) {
    super.onMeasure(widthMeasureSpec, heightMeasureSpec);
    int width = resolveSize(getSuggestedMinimumWidth(), widthMeasureSpec);
    int height = resolveSize(getSuggestedMinimumHeight(), heightMeasureSpec);
    setMeasuredDimension(width, height);

    int rotation = ((Activity) mContext).getWindowManager().getDefaultDisplay().getRotation();
    if ((Surface.ROTATION_0 == rotation || Surface.ROTATION_180 == rotation)) {//portrait
        mPreviewSize = getOptimalPreviewSize(mSupportedPreviewSizes, height, width);
    } else
        mPreviewSize = getOptimalPreviewSize(mSupportedPreviewSizes, width, height);//landscape

    if (mPreviewSize == null) return;
    float ratio;
    if (mPreviewSize.height >= mPreviewSize.width) {
        ratio = (float) mPreviewSize.height / (float) mPreviewSize.width;
    } else ratio = (float) mPreviewSize.width / (float) mPreviewSize.height;


    if (Build.VERSION.SDK_INT >= Build.VERSION_CODES.N && ((Activity) mContext).isInMultiWindowMode()) {
        if (getResources().getConfiguration().orientation == Configuration.ORIENTATION_PORTRAIT ||
                !(Surface.ROTATION_0 == rotation || Surface.ROTATION_180 == rotation)) {
            setMeasuredDimension(width, (int) (width / ratio));
        } else {
            setMeasuredDimension((int) (height / ratio), height);
        }
    } else {
        if ((Surface.ROTATION_0 == rotation || Surface.ROTATION_180 == rotation)) {
            Log.e("---", "onMeasure: " + height + " - " + width * ratio);
            //2264 - 2400.0 pix c -- yes
            //2240 - 2560.0 samsung -- yes
            //1582 - 1440.0 pix 2 -- no
            //1864 - 2048.0 sam tab -- yes
            //848 - 789.4737 iball -- no
            //1640 - 1600.0 nexus 7 -- no
            //1093 - 1066.6667 lenovo -- no
            //if width * ratio is > height, need to minus toolbar height
           if ((width * ratio) < height)
                setMeasuredDimension(width, (int) (width * ratio));
            else
                setMeasuredDimension(width, (int) (width * ratio) - toolbarHeight);
        } else {
            setMeasuredDimension((int) (height * ratio), height);
        }
    }
    requestLayout();
}

Bootstrap Datepicker - Months and Years Only

Why not call the $('.input-group.date').datepicker("remove"); when the select statement is changed then set your datepicker view then call the $('.input-group.date').datepicker("update");

How to implement my very own URI scheme on Android

Complementing the @DanielLew answer, to get the values of the parameteres you have to do this:

URI example: myapp://path/to/what/i/want?keyOne=valueOne&keyTwo=valueTwo

in your activity:

Intent intent = getIntent();
if (Intent.ACTION_VIEW.equals(intent.getAction())) {
  Uri uri = intent.getData();
  String valueOne = uri.getQueryParameter("keyOne");
  String valueTwo = uri.getQueryParameter("keyTwo");
}

Dictionary text file

http://www.math.sjsu.edu/~foster/dictionary.txt

350,000 words

Very late, but might be useful for others.

Why am I getting a FileNotFoundError?

Difficult to give code examples in the comments.

To read the words in the file, you can read the contents of the file, which gets you a string - this is what you were doing before, with the read() method - and then use split() to get the individual words. Split breaks up a String on the delimiter provided, or on whitespace by default. For example,

"the quick brown fox".split()

produces

['the', 'quick', 'brown', 'fox']

Similarly,

fileScan.read().split()

will give you an array of Strings. Hope that helps!

npm WARN enoent ENOENT: no such file or directory, open 'C:\Users\Nuwanst\package.json'

NOTE: if you are experiencing this issue in your CI pipeline, it is usually because npm runs npm ci instead of npm install. npm ci requires an accurate package-lock.json.

To fix this, whenever you are modifying packages in package.json (e.g. moving packages from devDependencies to Dependencies like I was doing) you should regenerate package-lock.json in your repository by running these commands locally, and then push the changes upstream:

rm -rf node_modules
npm install
git commit package-lock.json
git push

java.lang.RuntimeException: Unable to start activity ComponentInfo

After trying few answers they are either not related to my project or , I have tried cleaning and rebuilding (https://stackoverflow.com/a/48760966/8463813). But it didn't work for me directly. I have compared it with older version of code, in which i observed some library files(jars and aars in External Libraries directory) are missing. Tried Invalidate Cache and Restart worked, which created all the libraries and working fine.

WAMP shows error 'MSVCR100.dll' is missing when install

Today I installed Wamp server 3.0.6 (x64) on a Windows 10 machine with VS2017 installed. I had many Visual C++ Redistributable Packages installed, but not the 2012 one. Then I installed it and now I can run the Wamp server. You can find a download link here.

How to append contents of multiple files into one file

for i in {1..3}; do cat "$i.txt" >> 0.txt; done

I found this page because I needed to join 952 files together into one. I found this to work much better if you have many files. This will do a loop for however many numbers you need and cat each one using >> to append onto the end of 0.txt.

Edit:

as brought up in the comments:

cat {1..3}.txt >> 0.txt

or

cat {0..3}.txt >> all.txt

javascript regex for password containing at least 8 characters, 1 number, 1 upper and 1 lowercase

Using individual regular expressions to test the different parts would be considerably easier than trying to get one single regular expression to cover all of them. It also makes it easier to add or remove validation criteria.

Note, also, that your usage of .filter() was incorrect; it will always return a jQuery object (which is considered truthy in JavaScript). Personally, I'd use an .each() loop to iterate over all of the inputs, and report individual pass/fail statuses. Something like the below:

$(".buttonClick").click(function () {

    $("input[type=text]").each(function () {
        var validated =  true;
        if(this.value.length < 8)
            validated = false;
        if(!/\d/.test(this.value))
            validated = false;
        if(!/[a-z]/.test(this.value))
            validated = false;
        if(!/[A-Z]/.test(this.value))
            validated = false;
        if(/[^0-9a-zA-Z]/.test(this.value))
            validated = false;
        $('div').text(validated ? "pass" : "fail");
        // use DOM traversal to select the correct div for this input above
    });
});

Working demo

getting the X/Y coordinates of a mouse click on an image with jQuery

The below code works always even if any image makes the window scroll.

$(function() {
    $("#demo-box").click(function(e) {

      var offset = $(this).offset();
      var relativeX = (e.pageX - offset.left);
      var relativeY = (e.pageY - offset.top);

      alert("X: " + relativeX + "  Y: " + relativeY);

    });
});

Ref: http://css-tricks.com/snippets/jquery/get-x-y-mouse-coordinates/

Move view with keyboard using Swift

For Black Screen Error ( Swift 4 & 4.2 ) .

I fixed the black screen problem. In the verified solution The keyboard height changes after tapping and this is causing black screen.

Have to use UIKeyboardFrameEndUserInfoKey instead of UIKeyboardFrameBeginUserInfoKey

var isKeyboardAppear = false

override func viewDidLoad() {
    super.viewDidLoad() 
    NotificationCenter.default.addObserver(self, selector: #selector(keyboardWillShow), name: NSNotification.Name.UIKeyboardWillShow, object: nil)
    NotificationCenter.default.addObserver(self, selector: #selector(keyboardWillHide), name: NSNotification.Name.UIKeyboardWillHide, object: nil)
}

@objc func keyboardWillShow(notification: NSNotification) {
    if !isKeyboardAppear {
        if let keyboardSize = (notification.userInfo?[UIKeyboardFrameEndUserInfoKey] as? NSValue)?.cgRectValue {
            if self.view.frame.origin.y == 0{
                self.view.frame.origin.y -= keyboardSize.height
            }
        }
        isKeyboardAppear = true
    }
}

@objc func keyboardWillHide(notification: NSNotification) {
    if isKeyboardAppear {
        if let keyboardSize = (notification.userInfo?[UIKeyboardFrameEndUserInfoKey] as? NSValue)?.cgRectValue {
            if self.view.frame.origin.y != 0{
                self.view.frame.origin.y += keyboardSize.height
            }
        }
         isKeyboardAppear = false
    }
}

RadioGroup: How to check programmatically

Grab the radio group and look at the children to see if any are unchecked.

RadioGroup rg = (RadioGroup) view;
int checked = savedInstanceState.getInt(wrap.getAttributeName());

if(checked != -1) {
    RadioButton btn = (RadioButton) rg.getChildAt(checked);
    btn.toggle();
}

determine DB2 text string length

From similar question DB2 - find and compare the lentgh of the value in a table field - add RTRIM since LENGTH will return length of column definition. This should be correct:

select * from table where length(RTRIM(fieldName))=10

UPDATE 27.5.2019: maybe on older db2 versions the LENGTH function returned the length of column definition. On db2 10.5 I have tried the function and it returns data length, not column definition length:

select fieldname
, length(fieldName) len_only
, length(RTRIM(fieldName)) len_rtrim
from (values (cast('1234567890  ' as varchar(30)) )) 
as tab(fieldName)

FIELDNAME                      LEN_ONLY    LEN_RTRIM
------------------------------ ----------- -----------
1234567890                              12          10

One can test this by using this term:

where length(fieldName)!=length(rtrim(fieldName))

How to diff a commit with its parent?

Use git show $COMMIT. It'll show you the log message for the commit, and the diff of that particular commit.

How to remove last n characters from every element in the R vector

Although this is mostly the same with the answer by @nfmcclure, I prefer using stringr package as it provdies a set of functions whose names are most consistent and descriptive than those in base R (in fact I always google for "how to get the number of characters in R" as I can't remember the name nchar()).

library(stringr)
str_sub(iris$Species, end=-4)
#or 
str_sub(iris$Species, 1, str_length(iris$Species)-3)

This removes the last 3 characters from each value at Species column.

how to make a new line in a jupyter markdown cell

"We usually put ' (space)' after the first sentence before a new line, but it doesn't work in Jupyter."

That inspired me to try using two spaces instead of just one - and it worked!!

(Of course, that functionality could possibly have been introduced between when the question was asked in January 2017, and when my answer was posted in March 2018.)

Can I scroll a ScrollView programmatically in Android?

Yes, you can.

Let's say you got one Layout and inside that, you got many Views. So if you want to scroll to any View programmatically, you have to write the following code snippet:

For example:

content_main.xml

<ScrollView
    android:id="@+id/scrollView"
    android:layout_width="match_parent"
    android:layout_height="wrap_content">

    <LinearLayout
        android:layout_width="match_parent"
        android:layout_height="wrap_content"
        android:orientation="vertical">

        <Button
            android:id="@+id/btn"
            android:layout_width="match_parent"
            android:layout_height="wrap_content" />

        <TextView
            android:id="@+id/txtView"
            android:layout_width="match_parent"
            android:layout_height="wrap_content" />

    </LinearLayout>
</ScrollView>

MainActivity.java

ScrollView scrollView = (ScrollView) findViewById(R.id.scrollView);
Button btn = (Button) findViewById(R.id.ivEventBanner);
TextView txtView = (TextView) findViewById(R.id.ivEditBannerImage);

If you want to scroll to a specific View, let's say txtview, in this case, just write:

scrollView.smoothScrollTo(txtView.getScrollX(),txtView.getScrollY());

And you are done.

What is the difference between a definition and a declaration?

Find similar answers here: Technical Interview Questions in C.

A declaration provides a name to the program; a definition provides a unique description of an entity (e.g. type, instance, and function) within the program. Declarations can be repeated in a given scope, it introduces a name in a given scope.

A declaration is a definition unless:

  • Declaration declares a function without specifying its body,
  • Declaration contains an extern specifier and no initializer or function body,
  • Declaration is the declaration of a static class data member without a class definition,
  • Declaration is a class name definition,

A definition is a declaration unless:

  • Definition defines a static class data member,
  • Definition defines a non-inline member function.

Jquery .on('scroll') not firing the event while scrolling

$("body").on("custom-scroll", ".myDiv", function(){
    console.log("Scrolled :P");
})

$("#btn").on("click", function(){
    $("body").append('<div class="myDiv"><br><br><p>Content1<p><br><br><p>Content2<p><br><br></div>');
    listenForScrollEvent($(".myDiv"));
});


function listenForScrollEvent(el){
    el.on("scroll", function(){
        el.trigger("custom-scroll");
    })
}

see this post - Bind scroll Event To Dynamic DIV?

Creating a zero-filled pandas data frame

Assuming having a template DataFrame, which one would like to copy with zero values filled here...

If you have no NaNs in your data set, multiplying by zero can be significantly faster:

In [19]: columns = ["col{}".format(i) for i in xrange(3000)]                                                                                       

In [20]: indices = xrange(2000)

In [21]: orig_df = pd.DataFrame(42.0, index=indices, columns=columns)

In [22]: %timeit d = pd.DataFrame(np.zeros_like(orig_df), index=orig_df.index, columns=orig_df.columns)
100 loops, best of 3: 12.6 ms per loop

In [23]: %timeit d = orig_df * 0.0
100 loops, best of 3: 7.17 ms per loop

Improvement depends on DataFrame size, but never found it slower.

And just for the heck of it:

In [24]: %timeit d = orig_df * 0.0 + 1.0
100 loops, best of 3: 13.6 ms per loop

In [25]: %timeit d = pd.eval('orig_df * 0.0 + 1.0')
100 loops, best of 3: 8.36 ms per loop

But:

In [24]: %timeit d = orig_df.copy()
10 loops, best of 3: 24 ms per loop

EDIT!!!

Assuming you have a frame using float64, this will be the fastest by a huge margin! It is also able to generate any value by replacing 0.0 to the desired fill number.

In [23]: %timeit d = pd.eval('orig_df > 1.7976931348623157e+308 + 0.0')
100 loops, best of 3: 3.68 ms per loop

Depending on taste, one can externally define nan, and do a general solution, irrespective of the particular float type:

In [39]: nan = np.nan
In [40]: %timeit d = pd.eval('orig_df > nan + 0.0')
100 loops, best of 3: 4.39 ms per loop

How can I get the list of files in a directory using C or C++?

For a C only solution, please check this out. It only requires an extra header:

https://github.com/cxong/tinydir

tinydir_dir dir;
tinydir_open(&dir, "/path/to/dir");

while (dir.has_next)
{
    tinydir_file file;
    tinydir_readfile(&dir, &file);

    printf("%s", file.name);
    if (file.is_dir)
    {
        printf("/");
    }
    printf("\n");

    tinydir_next(&dir);
}

tinydir_close(&dir);

Some advantages over other options:

  • It's portable - wraps POSIX dirent and Windows FindFirstFile
  • It uses readdir_r where available, which means it's (usually) threadsafe
  • Supports Windows UTF-16 via the same UNICODE macros
  • It is C90 so even very ancient compilers can use it

Showing loading animation in center of page while making a call to Action method in ASP .NET MVC

I defined two functions in Site.Master:

    <script type="text/javascript">
    var spinnerVisible = false;
    function showProgress() {
        if (!spinnerVisible) {
            $("div#spinner").fadeIn("fast");
            spinnerVisible = true;
        }
    };
    function hideProgress() {
        if (spinnerVisible) {
            var spinner = $("div#spinner");
            spinner.stop();
            spinner.fadeOut("fast");
            spinnerVisible = false;
        }
    };
</script>

And special section:

    <div id="spinner">
        Loading...
    </div>

Visual style is defined in CSS:

div#spinner
{
    display: none;
    width:100px;
    height: 100px;
    position: fixed;
    top: 50%;
    left: 50%;
    background:url(spinner.gif) no-repeat center #fff;
    text-align:center;
    padding:10px;
    font:normal 16px Tahoma, Geneva, sans-serif;
    border:1px solid #666;
    margin-left: -50px;
    margin-top: -50px;
    z-index:2;
    overflow: auto;
}

How to disable Home and other system buttons in Android?

Frankly it is not possible to disable the home button at least on new api levels , that is from 4.0 onwards. It is also not advisable to do that. You can however, block the back button by overriding the

public void onBackPressed() {
    // do not call super onBackPressed.
}

in order to override the home button, you could use a timer for example, and after every time check if the main screen is your screen or not, or your package is on top or not, (i am sure you will get links to it), and display your activity using the flag single_top.

That way , even if the home button is pressed you will be able to bring your app to the top.

Also make sure that the app has a way to exit, because such kind of apps can really be annoying and should never be developed.

Happy coding.

P.S: It is not possible to intercept the home event, when the home button is pressed.

You can use on attach to window methods and also keyguard methods, but not for api levels from 4.0 onwards.

javascript createElement(), style problem

You need to call the appendChild function to append your new element to an existing element in the DOM.

In Subversion can I be a user other than my login name?

Most of the answers seem to be for svn+ssh, or don't seem to work for us.

For http access, the easiest way to log out an SVN user from the command line is:

rm ~/.subversion/auth/svn.simple/*

Hat tip: http://www.yolinux.com/TUTORIALS/Subversion.html

How to initialize private static members in C++?

int foo::i = 0; 

Is the correct syntax for initializing the variable, but it must go in the source file (.cpp) rather than in the header.

Because it is a static variable the compiler needs to create only one copy of it. You have to have a line "int foo:i" some where in your code to tell the compiler where to put it otherwise you get a link error. If that is in a header you will get a copy in every file that includes the header, so get multiply defined symbol errors from the linker.

Invalid Host Header when ngrok tries to connect to React dev server

If you use webpack devServer the simplest way is to set disableHostCheck, check webpack doc like this

devServer: {
    contentBase: path.join(__dirname, './dist'),
    compress: true,
    host: 'localhost',
    // host: '0.0.0.0',
    port: 8080,
    disableHostCheck: true //for ngrok
},

text-align: right; not working for <label>

Label is an inline element - so, unless a width is defined, its width is exact the same which the letters span. Your div element is a block element so its width is by default 100%.

You will have to place the text-align: right; on the div element in your case, or applying display: block; to your label

Another option is to set a width for each label and then use text-align. The display: block method will not be necessary using this.

Request UAC elevation from within a Python script?

This may not completely answer your question but you could also try using the Elevate Command Powertoy in order to run the script with elevated UAC privileges.

http://technet.microsoft.com/en-us/magazine/2008.06.elevation.aspx

I think if you use it it would look like 'elevate python yourscript.py'

Python conversion between coordinates

You can use the cmath module.

If the number is converted to a complex format, then it becomes easier to just call the polar method on the number.

import cmath
input_num = complex(1, 2) # stored as 1+2j
r, phi = cmath.polar(input_num)

Invalid hook call. Hooks can only be called inside of the body of a function component

I have just started using hooks and I got the above warning when i was calling useEffect inside a function:

Then I have to move the useEffect outside of the function as belows:

 const onChangeRetypePassword = async value => {
  await setRePassword(value);
//previously useEffect was here

  };
//useEffect outside of func 

 useEffect(() => {
  if (password !== rePassword) {
  setPasswdMismatch(true);

     } 
  else{
    setPasswdMismatch(false);
 }
}, [rePassword]);

Hope it will be helpful to someone !

Reset all the items in a form

Additional-> To clear the Child Controls The below function would clear the nested(Child) controls also, wrap up in a class.

 public static void ClearControl(Control control)
        {
            if (control is TextBox)
            {
                TextBox txtbox = (TextBox)control;
                txtbox.Text = string.Empty;
            }
            else if (control is CheckBox)
            {
                CheckBox chkbox = (CheckBox)control;
                chkbox.Checked = false;
            }
            else if (control is RadioButton)
            {
                RadioButton rdbtn = (RadioButton)control;
                rdbtn.Checked = false;
            }
            else if (control is DateTimePicker)
            {
                DateTimePicker dtp = (DateTimePicker)control;
                dtp.Value = DateTime.Now;
            }
            else if (control is ComboBox)
            {
                ComboBox cmb = (ComboBox)control;
                if (cmb.DataSource != null)
                {
                    cmb.SelectedItem = string.Empty;
                    cmb.SelectedValue = 0;
                }
            }
            ClearErrors(control);
            // repeat for combobox, listbox, checkbox and any other controls you want to clear
            if (control.HasChildren)
            {
                foreach (Control child in control.Controls)
                {
                    ClearControl(child);
                }
            }


        }

ValueError: unsupported format character while forming strings

For anyone checking this using python 3:

If you want to print the following output "100% correct":

python 3.8: print("100% correct")
python 3.7 and less: print("100%% correct")


A neat programming workaround for compatibility across diff versions of python is shown below:

Note: If you have to use this, you're probably experiencing many other errors... I'd encourage you to upgrade / downgrade python in relevant machines so that they are all compatible.

DevOps is a notable exception to the above -- implementing the following code would indeed be appropriate for specific DevOps / Debugging scenarios.

import sys

if version_info.major==3:
    if version_info.minor>=8:
        my_string = "100% correct"
    else:
        my_string = "100%% correct"

# Finally
print(my_string)

How to add new contacts in android

This is working fine for me:

ArrayList<ContentProviderOperation> ops = new ArrayList<ContentProviderOperation>();
            int rawContactInsertIndex = ops.size();

            ops.add(ContentProviderOperation.newInsert(RawContacts.CONTENT_URI)
                    .withValue(RawContacts.ACCOUNT_TYPE, null)
                    .withValue(RawContacts.ACCOUNT_NAME, null).build());
            ops.add(ContentProviderOperation
                    .newInsert(Data.CONTENT_URI)
                    .withValueBackReference(Data.RAW_CONTACT_ID,rawContactInsertIndex)
                    .withValue(Data.MIMETYPE, StructuredName.CONTENT_ITEM_TYPE)
                    .withValue(StructuredName.DISPLAY_NAME, "Vikas Patidar") // Name of the person
                    .build());
            ops.add(ContentProviderOperation
                    .newInsert(Data.CONTENT_URI)
                    .withValueBackReference(
                            ContactsContract.Data.RAW_CONTACT_ID,   rawContactInsertIndex)
                    .withValue(Data.MIMETYPE, Phone.CONTENT_ITEM_TYPE)
                    .withValue(Phone.NUMBER, "9999999999") // Number of the person
                    .withValue(Phone.TYPE, Phone.TYPE_MOBILE).build()); // Type of mobile number                    
            try
            {
                ContentProviderResult[] res = getContentResolver().applyBatch(ContactsContract.AUTHORITY, ops);
            }
            catch (RemoteException e)
            { 
                // error
            }
            catch (OperationApplicationException e) 
            {
                // error
            }       

What's the Kotlin equivalent of Java's String[]?

To create an empty Array of Strings in Kotlin you should use one of the following six approaches:

First approach:

val empty = arrayOf<String>()

Second approach:

val empty = arrayOf("","","")

Third approach:

val empty = Array<String?>(3) { null }

Fourth approach:

val empty = arrayOfNulls<String>(3)

Fifth approach:

val empty = Array<String>(3) { "it = $it" }

Sixth approach:

val empty = Array<String>(0, { _ -> "" })

Selecting distinct values from a JSON

First we can just run map() function to get the new array with the results of calling a provided function on every element in the varjson.DATA.

varjson.DATA.map(({name})=>name))

After getting the array of name from the varjson.DATA. We can convert it into a set that will discard all duplicate entries of array and apply spread operator to get a array of unique names:

[...new Set(varjson.DATA.map(({name})=>name))]

_x000D_
_x000D_
const varjson = {_x000D_
  "DATA": [{_x000D_
      "id": 11,_x000D_
      "name": "ajax",_x000D_
      "subject": "OR",_x000D_
      "mark": 63_x000D_
    },_x000D_
    {_x000D_
      "id": 12,_x000D_
      "name": "javascript",_x000D_
      "subject": "OR",_x000D_
      "mark": 63_x000D_
    },_x000D_
    {_x000D_
      "id": 13,_x000D_
      "name": "jquery",_x000D_
      "subject": "OR",_x000D_
      "mark": 63_x000D_
    },_x000D_
    {_x000D_
      "id": 14,_x000D_
      "name": "ajax",_x000D_
      "subject": "OR",_x000D_
      "mark": 63_x000D_
    },_x000D_
    {_x000D_
      "id": 15,_x000D_
      "name": "jquery",_x000D_
      "subject": "OR",_x000D_
      "mark": 63_x000D_
    },_x000D_
    {_x000D_
      "id": 16,_x000D_
      "name": "ajax",_x000D_
      "subject": "OR",_x000D_
      "mark": 63_x000D_
    },_x000D_
    {_x000D_
      "id": 20,_x000D_
      "name": "ajax",_x000D_
      "subject": "OR",_x000D_
      "mark": 63_x000D_
    }_x000D_
  ],_x000D_
  "COUNT": "120"_x000D_
}_x000D_
_x000D_
console.log( [...new Set(varjson.DATA.map(({name})=>name))]);
_x000D_
_x000D_
_x000D_

How to debug Angular JavaScript Code

var rootEle = document.querySelector("html");
var ele = angular.element(rootEle); 

scope() We can fetch the $scope from the element (or its parent) by using the scope() method on the element:

var scope = ele.scope();

injector()

var injector = ele.injector();

With this injector, we can then then instantiate any Angular object inside of our app, such as services, other controllers, or any other object

SQL Server convert string to datetime

For instance you can use

update tablename set datetimefield='19980223 14:23:05'
update tablename set datetimefield='02/23/1998 14:23:05'
update tablename set datetimefield='1998-12-23 14:23:05'
update tablename set datetimefield='23 February 1998 14:23:05'
update tablename set datetimefield='1998-02-23T14:23:05'

You need to be careful of day/month order since this will be language dependent when the year is not specified first. If you specify the year first then there is no problem; date order will always be year-month-day.

How to get the current branch name in Git?

Well simple enough, I got it in a one liner (bash)

git branch | sed -n '/\* /s///p'

(credit: Limited Atonement)

And while I am there, the one liner to get the remote tracking branch (if any)

git rev-parse --symbolic-full-name --abbrev-ref @{u}

How to convert a set to a list in python?

Whenever you are stuck in such type of problems, try to find the datatype of the element you want to convert first by using :

type(my_set)

Then, Use:

  list(my_set) 

to convert it to a list. You can use the newly built list like any normal list in python now.

What does /p mean in set /p?

For future reference, you can get help for any command by using the /? switch, which should explain what switches do what.

According to the set /? screen, the format for set /p is SET /P variable=[promptString] which would indicate that the p in /p is "prompt." It just prints in your example because <nul passes in a nul character which immediately ends the prompt so it just acts like it's printing. It's still technically prompting for input, it's just immediately receiving it.

/L in for /L generates a List of numbers.

From ping /?:

Usage: ping [-t] [-a] [-n count] [-l size] [-f] [-i TTL] [-v TOS]
            [-r count] [-s count] [[-j host-list] | [-k host-list]]
            [-w timeout] [-R] [-S srcaddr] [-4] [-6] target_name

Options:
    -t             Ping the specified host until stopped.
                   To see statistics and continue - type Control-Break;
                   To stop - type Control-C.
    -a             Resolve addresses to hostnames.
    -n count       Number of echo requests to send.
    -l size        Send buffer size.
    -f             Set Don't Fragment flag in packet (IPv4-only).
    -i TTL         Time To Live.
    -v TOS         Type Of Service (IPv4-only. This setting has been deprecated
                   and has no effect on the type of service field in the IP Header).
    -r count       Record route for count hops (IPv4-only).
    -s count       Timestamp for count hops (IPv4-only).
    -j host-list   Loose source route along host-list (IPv4-only).
    -k host-list   Strict source route along host-list (IPv4-only).
    -w timeout     Timeout in milliseconds to wait for each reply.
    -R             Use routing header to test reverse route also (IPv6-only).
    -S srcaddr     Source address to use.
    -4             Force using IPv4.
    -6             Force using IPv6.

Compiling simple Hello World program on OS X via command line

Use the following for multiple .cpp files

g++ *.cpp
./a.out

The executable gets signed with invalid entitlements in Xcode

Simple clean-and-build seemed to fix it for me.

Extract specific columns from delimited file using Awk

Tabulator is a set of unix command line tools to work with csv files that have header lines. Here is an example to extract columns by name from a file test.csv:

name,sex,house_nr,height,shoe_size
arthur,m,42,181,11.5
berta,f,101,163,8.5
chris,m,1333,175,10
don,m,77,185,12.5
elisa,f,204,166,7

Then tblmap -k name,height test.csv produces

name,height
arthur,181
berta,163
chris,175
don,185
elisa,166

JavaScript window resize event

var EM = new events_managment();

EM.addEvent(window, 'resize', function(win,doc, event_){
    console.log('resized');
    //EM.removeEvent(win,doc, event_);
});

function events_managment(){
    this.events = {};
    this.addEvent = function(node, event_, func){
        if(node.addEventListener){
            if(event_ in this.events){
                node.addEventListener(event_, function(){
                    func(node, event_);
                    this.events[event_](win_doc, event_);
                }, true);
            }else{
                node.addEventListener(event_, function(){
                    func(node, event_);
                }, true);
            }
            this.events[event_] = func;
        }else if(node.attachEvent){

            var ie_event = 'on' + event_;
            if(ie_event in this.events){
                node.attachEvent(ie_event, function(){
                    func(node, ie_event);
                    this.events[ie_event]();
                });
            }else{
                node.attachEvent(ie_event, function(){
                    func(node, ie_event);
                });
            }
            this.events[ie_event] = func;
        }
    }
    this.removeEvent = function(node, event_){
        if(node.removeEventListener){
            node.removeEventListener(event_, this.events[event_], true);
            this.events[event_] = null;
            delete this.events[event_];
        }else if(node.detachEvent){
            node.detachEvent(event_, this.events[event_]);
            this.events[event_] = null;
            delete this.events[event_];
        }
    }
}

How to tell if UIViewController's view is visible

XCode 6.4, for iOS 8.4, ARC enabled

Obviously lots of ways of doing it. The one that has worked for me is the following...

@property(nonatomic, readonly, getter=isKeyWindow) BOOL keyWindow

This can be used in any view controller in the following way,

[self.view.window isKeyWindow]

If you call this property in -(void)viewDidLoad you get 0, then if you call this after -(void)viewDidAppear:(BOOL)animated you get 1.

Hope this helps someone. Thanks! Cheers.

How to get length of a string using strlen function

#include<iostream>
#include<conio.h>
#include<string.h>
using namespace std;
int main()

{
    char str[80];

    int i;

    cout<<"\n enter string:";

    cin.getline(str,80);

    int n=strlen(str);

    cout<<"\n lenght is:"<<n;

    getch();

    return 0;

}

This is the program if you want to use strlen . Hope this helps!

How to generate and manually insert a uniqueidentifier in sql server?

Kindly check Column ApplicationId datatype in Table aspnet_Users , ApplicationId column datatype should be uniqueidentifier .

*Your parameter order is passed wrongly , Parameter @id should be passed as first argument, but in your script it is placed in second argument..*

So error is raised..

Please refere sample script:

DECLARE @id uniqueidentifier
SET @id = NEWID()
Create Table #temp1(AppId uniqueidentifier)

insert into #temp1 values(@id)

Select * from #temp1

Drop Table #temp1

How to do an array of hashmaps?

You can use something like this:

import java.util.ArrayList;
import java.util.HashMap;
import java.util.List;
import java.util.Map;


public class testHashes {

public static void main(String args[]){
    Map<String,String> myMap1 = new HashMap<String, String>();

    List<Map<String , String>> myMap  = new ArrayList<Map<String,String>>();

    myMap1.put("URL", "Val0");
    myMap1.put("CRC", "Vla1");
    myMap1.put("SIZE", "Val2");
    myMap1.put("PROGRESS", "Val3");

    myMap.add(0,myMap1);
    myMap.add(1,myMap1);

    for (Map<String, String> map : myMap) {
        System.out.println(map.get("URL"));
        System.out.println(map.get("CRC"));
        System.out.println(map.get("SIZE"));
        System.out.println(map.get("PROGRESS"));
    }

    //System.out.println(myMap);

}


}

Creating a static class with no instances

The Pythonic way to create a static class is simply to declare those methods outside of a class (Java uses classes both for objects and for grouping related functions, but Python modules are sufficient for grouping related functions that do not require any object instance). However, if you insist on making a method at the class level that doesn't require an instance (rather than simply making it a free-standing function in your module), you can do so by using the "@staticmethod" decorator.

That is, the Pythonic way would be:

# My module
elements = []

def add_element(x):
  elements.append(x)

But if you want to mirror the structure of Java, you can do:

# My module
class World(object):
  elements = []

  @staticmethod
  def add_element(x):
    World.elements.append(x)

You can also do this with @classmethod if you care to know the specific class (which can be handy if you want to allow the static method to be inherited by a class inheriting from this class):

# My module
class World(object):
  elements = []

  @classmethod
  def add_element(cls, x):
    cls.elements.append(x)

How to get value from form field in django framework?

Using a form in a view pretty much explains it.

The standard pattern for processing a form in a view looks like this:

def contact(request):
    if request.method == 'POST': # If the form has been submitted...
        form = ContactForm(request.POST) # A form bound to the POST data
        if form.is_valid(): # All validation rules pass
            # Process the data in form.cleaned_data
            # ...

            print form.cleaned_data['my_form_field_name']

            return HttpResponseRedirect('/thanks/') # Redirect after POST
    else:
        form = ContactForm() # An unbound form

    return render_to_response('contact.html', {
        'form': form,
    })

What is the difference between a symbolic link and a hard link?

Also:

  1. Read performance of hard links is better than symbolic links (micro-performance)
  2. Symbolic links can be copied, version-controlled, ..etc. In another words, they are an actual file. On the other end, a hard link is something at a slightly lower level and you will find that compared to symbolic links, there are less tools that provide means for working with the hard links as hard links and not as normal files

How to check if a file exists before creating a new file

you can also use Boost.

 boost::filesystem::exists( filename );

it works for files and folders.

And you will have an implementation close to something ready for C++14 in which filesystem should be part of the STL (see here).

How add "or" in switch statements?

Case-statements automatically fall through if you don't specify otherwise (by writing break). Therefor you can write

switch(myvar)
{
   case 2:
   case 5:
   {
      //your code
   break;
   }

// etc... }

How to check whether a Storage item is set?

Best and Safest way i can suggest is this,

if(Object.prototype.hasOwnProperty.call(localStorage, 'infiniteScrollEnabled')){
    // init variable/set default variable for item
    localStorage.setItem("infiniteScrollEnabled", true);
}

This passes through ESLint's no-prototype-builtins rule.

Insert an element at a specific index in a list and return the updated list

l.insert(index, obj) doesn't actually return anything. It just updates the list.

As ATO said, you can do b = a[:index] + [obj] + a[index:]. However, another way is:

a = [1, 2, 4]
b = a[:]
b.insert(2, 3)

COPY with docker but with exclusion

FOR A ONE LINER SOLUTION, type the following in Command prompt or Terminal at project root.

echo node_modules > .dockerignore

This creates the extension-less . prefixed file without any issue. Replace node_modules with the folder you want to exclude.

How to pass parameters to $http in angularjs?

Here is how you do it:

$http.get("/url/to/resource/", {params:{"param1": val1, "param2": val2}})
    .then(function (response) { /* */ })...

Angular takes care of encoding the parameters.

Maxim Shoustin's answer does not work ({method:'GET', url:'/search', jsonData} is not a valid JavaScript literal) and JeyTheva's answer, although simple, is dangerous as it allows XSS (unsafe values are not escaped when you concatenate them).

Add a background image to shape in XML Android

I used the following for a drawable image with a circular background.

<?xml version="1.0" encoding="utf-8"?>
<layer-list xmlns:android="http://schemas.android.com/apk/res/android">
    <item>
        <shape android:shape="oval">
            <solid android:color="@color/colorAccent"/>
        </shape>
    </item>
    <item
        android:drawable="@drawable/ic_select"
        android:bottom="20dp"
        android:left="20dp"
        android:right="20dp"
        android:top="20dp"/>
</layer-list>

Here is what it looks like

enter image description here

Hope that helps someone out.

How do I find the stack trace in Visual Studio?

Using the Call Stack Window

To open the Call Stack window in Visual Studio, from the Debug menu, choose Windows>Call Stack. To set the local context to a particular row in the stack trace display, double click the first column of the row.

http://msdn.microsoft.com/en-us/library/windows/hardware/hh439516(v=vs.85).aspx

Making the main scrollbar always visible

body { 
    min-height: 101vh; 
} 

works the best for me

How to set selectedIndex of select element using display text?

Try this:

function SelectAnimal() {
    var sel = document.getElementById('Animals');
    var val = document.getElementById('AnimalToFind').value;
    for(var i = 0, j = sel.options.length; i < j; ++i) {
        if(sel.options[i].innerHTML === val) {
           sel.selectedIndex = i;
           break;
        }
    }
}

Substitute a comma with a line break in a cell

You can also do this without VBA from the find/replace dialogue box. My answer was at https://stackoverflow.com/a/6116681/509840 .

Detect element content changes with jQuery

These are mutation events.

I have not used mutation event APIs in jQuery, but a cursory search led me to this project on GitHub. I am unaware of the project's maturity.

How do I clear my Jenkins/Hudson build history?

Using Script Console.

In case the jobs are grouped it's possible to either give it a full name with forward slashes:

getItemByFullName("folder_name/job_name") 
job.getBuilds().each { it.delete() }
job.nextBuildNumber = 1
job.save()

or traverse the hierarchy like this:

def folder = Jenkins.instance.getItem("folder_name")
def job = folder.getItem("job_name")
job.getBuilds().each { it.delete() }
job.nextBuildNumber = 1
job.save()

Python + Django page redirect

page_path = define in urls.py

def deletePolls(request):
    pollId = deletePool(request.GET['id'])
    return HttpResponseRedirect("/page_path/")

Reduce git repository size

In my case, I pushed several big (> 100Mb) files and then proceeded to remove them. But they were still in the history of my repo, so I had to remove them from it as well.

What did the trick was:

bfg -b 100M  # To remove all blobs from history, whose size is superior to 100Mb
git reflog expire --expire=now --all
git gc --prune=now --aggressive

Then, you need to push force on your branch:

git push origin <your_branch_name> --force

Note: bfg is a tool that can be installed on Linux and macOS using brew:

brew install bfg

Increase bootstrap dropdown menu width

Update 2018

You should be able to just set it using CSS like this..

.dropdown-menu {
  min-width:???px;
}

This works in both Bootstrap 3 and Bootstrap 4.0.0 (demo).


A no extra CSS option in Bootstrap 4 is using the sizing utils to change the width. For example, here the w-100 (width:100%) class is used for the dropdown menu to fill the width of it's parent....

 <ul class="dropdown-menu w-100">
      <li><a class="nav-link" href="#">Choice1</a></li>
      <li><a class="nav-link" href="#">Choice2</a></li>
      <li><a class="nav-link" href="#">Choice3</a></li>
 </ul>

https://www.codeply.com/go/pAqaPj59N0

How do I pull files from remote without overwriting local files?

You can stash your local changes first, then pull, then pop the stash.

git stash
git pull origin master
git stash pop

Anything that overrides changes from remote will have conflicts which you will have to manually resolve.

Optimistic vs. Pessimistic locking

One use case for optimistic locking is to have your application use the database to allow one of your threads / hosts to 'claim' a task. This is a technique that has come in handy for me on a regular basis.

The best example I can think of is for a task queue implemented using a database, with multiple threads claiming tasks concurrently. If a task has status 'Available', 'Claimed', 'Completed', a db query can say something like "Set status='Claimed' where status='Available'. If multiple threads try to change the status in this way, all but the first thread will fail because of dirty data.

Note that this is a use case involving only optimistic locking. So as an alternative to saying "Optimistic locking is used when you don't expect many collisions", it can also be used where you expect collisions but want exactly one transaction to succeed.

Allow click on twitter bootstrap dropdown toggle link?

Since there is not really an answer that works (selected answer disables dropdown), or overrides using javascript, here goes.

This is all html and css fix (uses two <a> tags):

<ul class="nav">
 <li class="dropdown dropdown-li">
    <a class="dropdown-link" href="http://google.com">Dropdown</a>
    <a class="dropdown-caret dropdown-toggle"><b class="caret"></b></a>
    <ul class="dropdown-menu">
        <li><a href="#">Link 1</a></li>
        <li><a href="#">Link 2</a></li>
    </ul>
 </li>
</ul>

Now here's the CSS you need.

.dropdown-li {
    display:inline-block !important;
}
.dropdown-link {
    display:inline-block !important; 
    padding-right:4px !important;
}
.dropdown-caret {
    display:inline-block !important; 
    padding-left:4px !important;
}

Assuming you will want the both <a> tags to highlight on hover of either one, you will also need to override bootstrap, you might play around with the following:

.nav > li:hover {
    background-color: #f67a47; /*hover background color*/
}
.nav > li:hover > a {
    color: white; /*hover text color*/
}
.nav > li:hover > ul > a {
    color: black; /*dropdown item text color*/
}

Could not load file or assembly 'System.Web.WebPages.Razor, Version=2.0.0.0

I uninstalled ASP.NET MVC 4 using the Windows Control Panel, then reinstalled it by running AspNetMVC4Setup.exe (which I got from https://www.microsoft.com/en-us/download/details.aspx?id=30683), and that fixed the issue for me.

In other words, I didn't need to use Nuget or Visual Studio.

Function to close the window in Tkinter

def quit(self):
    self.root.destroy()

Add parentheses after destroy to call the method.

When you use command=self.root.destroy you pass the method to Tkinter.Button without the parentheses because you want Tkinter.Button to store the method for future calling, not to call it immediately when the button is created.

But when you define the quit method, you need to call self.root.destroy() in the body of the method because by then the method has been called.

Better way of getting time in milliseconds in javascript?

I know this is a pretty old thread, but to keep things up to date and more relevant, you can use the more accurate performance.now() functionality to get finer grain timing in javascript.

window.performance = window.performance || {};
performance.now = (function() {
    return performance.now       ||
        performance.mozNow    ||
        performance.msNow     ||
        performance.oNow      ||
        performance.webkitNow ||            
        Date.now  /*none found - fallback to browser default */
})();

How to write text on a image in windows using python opencv2

I had a similar problem. I would suggest using the PIL library in python as it draws the text in any given font, compared to limited fonts in OpenCV. With PIL you can choose any font installed on your system.

from PIL import ImageFont, ImageDraw, Image
import numpy as np
import cv2

image = cv2.imread("lena.png")

# Convert to PIL Image
cv2_im_rgb = cv2.cvtColor(image, cv2.COLOR_BGR2RGB)
pil_im = Image.fromarray(cv2_im_rgb)

draw = ImageDraw.Draw(pil_im)

# Choose a font
font = ImageFont.truetype("Roboto-Regular.ttf", 50)

# Draw the text
draw.text((0, 0), "Your Text Here", font=font)

# Save the image
cv2_im_processed = cv2.cvtColor(np.array(pil_im), cv2.COLOR_RGB2BGR)
cv2.imwrite("result.png", cv2_im_processed)

result.png

Grep for beginning and end of line?

It should be noted that not only will the caret (^) behave differently within the brackets, it will have the opposite result of placing it outside of the brackets. Placing the caret where you have it will search for all strings NOT beginning with the content you placed within the brackets. You also would want to place a period before the asterisk in between your brackets as with grep, it also acts as a "wildcard".

grep ^[.rwx].*[0-9]$

This should work for you, I noticed that some posters used a character class in their expressions which is an effective method as well, but you were not using any in your original expression so I am trying to get one as close to yours as possible explaining every minor change along the way so that it is better understood. How can we learn otherwise?

How to create id with AUTO_INCREMENT on Oracle?

Assuming you mean a column like the SQL Server identity column?

In Oracle, you use a SEQUENCE to achieve the same functionality. I'll see if I can find a good link and post it here.

Update: looks like you found it yourself. Here is the link anyway: http://www.techonthenet.com/oracle/sequences.php

Creating composite primary key in SQL Server

If you use management studio, simply select the wardNo, BHTNo, testID columns and click on the key mark in the toolbar.

enter image description here

Command for this is,

ALTER TABLE dbo.testRequest
ADD CONSTRAINT PK_TestRequest 
PRIMARY KEY (wardNo, BHTNo, TestID)

Plot different DataFrames in the same figure

Although Chang's answer explains how to plot multiple times on the same figure, in this case you might be better off in this case using a groupby and unstacking:

(Assuming you have this in dataframe, with datetime index already)

In [1]: df
Out[1]:
            value  
datetime                         
2010-01-01      1  
2010-02-01      1  
2009-01-01      1  

# create additional month and year columns for convenience
df['Month'] = map(lambda x: x.month, df.index)
df['Year'] = map(lambda x: x.year, df.index)    

In [5]: df.groupby(['Month','Year']).mean().unstack()
Out[5]:
       value      
Year    2009  2010
Month             
1          1     1
2        NaN     1

Now it's easy to plot (each year as a separate line):

df.groupby(['Month','Year']).mean().unstack().plot()

Flask at first run: Do not use the development server in a production environment

Unless you tell the development server that it's running in development mode, it will assume you're using it in production and warn you not to. The development server is not intended for use in production. It is not designed to be particularly efficient, stable, or secure.

Enable development mode by setting the FLASK_ENV environment variable to development.

$ export FLASK_APP=example
$ export FLASK_ENV=development
$ flask run

If you're running in PyCharm (or probably any other IDE) you can set environment variables in the run configuration.

Development mode enables the debugger and reloader by default. If you don't want these, pass --no-debugger or --no-reloader to the run command.


That warning is just a warning though, it's not an error preventing your app from running. If your app isn't working, there's something else wrong with your code.

How to make the web page height to fit screen height

As another guy described here, all you need to do is add

height: 100vh;

to the style of whatever you need to fill the screen

Python: How to get stdout after running os.system?

I had to use os.system, since subprocess was giving me a memory error for larger tasks. Reference for this problem here. So, in order to get the output of the os.system command I used this workaround:

import os

batcmd = 'dir'
result_code = os.system(batcmd + ' > output.txt')
if os.path.exists('output.txt'):
    fp = open('output.txt', "r")
    output = fp.read()
    fp.close()
    os.remove('output.txt')
    print(output)

Convert a number into a Roman Numeral in javaScript

function convertToRoman(num) {

  var search = {
    "0":["I","II","III","IV","V","VI","VII","VIII","IX"],
    "1":["X","XX","XXX","XL","L","LX","LXX","LXXX","XC"],
    "2":["C","CC","CCC","CD","D","DC","DCC","DCCC","CM"],
    "3":["M","MM","MMM","MV^","V^","V^M","V^MM","V^MMM","MX^"],
  };

  var numArr = num.toString().split("").reverse();
  var romanReturn = [];
  for(var i=0; i<numArr.length; i++){
    romanReturn.unshift(search[i][numArr[i]-1]);
  }
  return romanReturn.join("");
}

Passing A List Of Objects Into An MVC Controller Method Using jQuery Ajax

I am using a .Net Core 2.1 Web Application and could not get a single answer here to work. I either got a blank parameter (if the method was called at all) or a 500 server error. I started playing with every possible combination of answers and finally got a working result.

In my case the solution was as follows:

Script - stringify the original array (without using a named property)

    $.ajax({
        type: 'POST',
        contentType: 'application/json; charset=utf-8',
        url: mycontrolleraction,
        data: JSON.stringify(things)
    });

And in the controller method, use [FromBody]

    [HttpPost]
    public IActionResult NewBranch([FromBody]IEnumerable<Thing> things)
    {
        return Ok();
    }

Failures include:

  • Naming the content

    data: { content: nodes }, // Server error 500

  • Not having the contentType = Server error 500

Notes

  • dataType is not needed, despite what some answers say, as that is used for the response decoding (so not relevant to the request examples here).
  • List<Thing> also works in the controller method

jQuery - get all divs inside a div with class ".container"

From http://api.jquery.com/jQuery/

Selector Context By default, selectors perform their searches within the DOM starting at the document root. However, an alternate context can be given for the search by using the optional second parameter to the $() function. For example, to do a search within an event handler, the search can be restricted like so:

$( "div.foo" ).click(function() { 
   $( "span", this ).addClass( "bar" );
});

When the search for the span selector is restricted to the context of this, only spans within the clicked element will get the additional class.

So for your example I would suggest something like:

$("div", ".container").each(function(){
     //do whatever
 });

How to click on hidden element in Selenium WebDriver?

overflow:hidden 

does not always mean that the element is hidden or non existent in the DOM, it means that the overflowing chars that do not fit in the element are being trimmed. Basically it means that do not show scrollbar even if it should be showed, so in your case the link with text

Plastic Spiral Bind

could possibly be shown as "Plastic Spir..." or similar. So it is possible, that this linkText indeed is non existent.

So you can probably try:

driver.findElement(By.partialLinkText("Plastic ")).click();

or xpath:

//a[contains(@title, \"Plastic Spiral Bind\")]

What’s the best way to reload / refresh an iframe?

If you tried all of the other suggestions, and couldn't get any of them to work (like I couldn't), here's something you can try that may be useful.

HTML

<a class="refresh-this-frame" rel="#iframe-id-0">Refresh</a>
<iframe src="" id="iframe-id-0"></iframe>

JS

$('.refresh-this-frame').click(function() {
    var thisIframe = $(this).attr('rel');
    var currentState = $(thisIframe).attr('src');
    function removeSrc() {
        $(thisIframe).attr('src', '');
    }
    setTimeout (removeSrc, 100);
    function replaceSrc() {
        $(thisIframe).attr('src', currentState);
    }
    setTimeout (replaceSrc, 200);
});

I initially set out to try and save some time with RWD and cross-browser testing. I wanted to create a quick page that housed a bunch of iframes, organized into groups that I would show/hide at will. Logically you'd want to be able to easily and quickly refresh any given frame.

I should note that the project I am working on currently, the one in use in this test-bed, is a one-page site with indexed locations (e.g. index.html#home). That may have had something to do with why I couldn't get any of the other solutions to refresh my particular frame.

Having said that, I know it's not the cleanest thing in the world, but it works for my purposes. Hope this helps someone. Now if only I could figure out how to keep the iframe from scrolling the parent page each time there's animation inside iframe...

EDIT: I realized that this doesn't "refresh" the iframe like I'd hoped it would. It will reload the iframe's initial source though. Still can't figure out why I couldn't get any of the other options to work..

UPDATE: The reason I couldn't get any of the other methods to work is because I was testing them in Chrome, and Chrome won't allow you to access an iframe's content (Explanation: Is it likely that future releases of Chrome support contentWindow/contentDocument when iFrame loads a local html file from local html file?) if it doesn't originate from the same location (so far as I understand it). Upon further testing, I can't access contentWindow in FF either.

AMENDED JS

$('.refresh-this-frame').click(function() {
    var targetID = $(this).attr('rel');
    var targetSrc = $(targetID).attr('src');
    var cleanID = targetID.replace("#","");     
    var chromeTest = ( navigator.userAgent.match(/Chrome/g) ? true : false );
    var FFTest = ( navigator.userAgent.match(/Firefox/g) ? true : false );      
    if (chromeTest == true) {
        function removeSrc() {
            $(targetID).attr('src', '');
        }
        setTimeout (removeSrc, 100);
        function replaceSrc() {
            $(targetID).attr('src', targetSrc);
        }
        setTimeout (replaceSrc, 200);
    }
    if (FFTest == true) {
        function removeSrc() {
            $(targetID).attr('src', '');
        }
        setTimeout (removeSrc, 100);
        function replaceSrc() {
            $(targetID).attr('src', targetSrc);
        }
        setTimeout (replaceSrc, 200);
    }       
    if (chromeTest == false && FFTest == false) {
        var targetLoc = (document.getElementById(cleanID).contentWindow.location).toString();
        function removeSrc() {
            $(targetID).attr('src', '');
        }
        setTimeout (removeSrc, 100);
        function replaceSrc2() {
            $(targetID).attr('src', targetLoc);
        }
        setTimeout (replaceSrc2, 200);
    }
});

System.MissingMethodException: Method not found?

I just ran into this on a .NET MVC project. The root cause was conflicting versions of NuGet packages. I had a solution with several projects. Each of the projects had some NuGet packages. In one project I had a version of the Enterprise Library Semantic Logging package, and in two other projects (that reference the first) I had older versions of the same package. It all compiles without error, but it gave a mysterious "Method not found" error when I tried to use the package.

The fix was to remove the old NuGet packages from the two projects, so that it was only included in the one project that actually needed it. (Also I did a clean rebuild of the whole solution.)

Bootstrap dropdown menu not working (not dropping down when clicked)

Just Remove the type="text/javascript"

<script src="JavaScript/jquery.js" />
<script src="JavaScript/bootstrap-min.js" />

Here is the update - http://jsfiddle.net/andieje/kRX6n/

Force drop mysql bypassing foreign key constraint

You can use the following steps, its worked for me to drop table with constraint,solution already explained in the above comment, i just added screen shot for that -enter image description here

Running JAR file on Windows

Easiest route is probably upgrading or re-installing the Java Runtime Environment (JRE).

Or this:

  • Open the Windows Explorer, from the Tools select 'Folder Options...'
  • Click the File Types tab, scroll down and select JAR File type.
  • Press the Advanced button.
  • In the Edit File Type dialog box, select open in Actions box and click Edit...
  • Press the Browse button and navigate to the location the Java interpreter javaw.exe.
  • In the Application used to perform action field, needs to display something similar to C:\Program Files\Java\j2re1.4.2_04\bin\javaw.exe" -jar "%1" % (Note: the part starting with 'javaw' must be exactly like that; the other part of the path name can vary depending on which version of Java you're using) then press the OK buttons until all the dialogs are closed.

Which was stolen from here: http://windowstipoftheday.blogspot.com/2005/10/setting-jar-file-association.html

Configuring so that pip install can work from github

I had similar issue when I had to install from github repo, but did not want to install git , etc.

The simple way to do it is using zip archive of the package. Add /zipball/master to the repo URL:

    $ pip install https://github.com/hmarr/django-debug-toolbar-mongo/zipball/master
Downloading/unpacking https://github.com/hmarr/django-debug-toolbar-mongo/zipball/master
  Downloading master
  Running setup.py egg_info for package from https://github.com/hmarr/django-debug-toolbar-mongo/zipball/master
Installing collected packages: django-debug-toolbar-mongo
  Running setup.py install for django-debug-toolbar-mongo
Successfully installed django-debug-toolbar-mongo
Cleaning up...

This way you will make pip work with github source repositories.

Get difference between two lists

You can cycle through the first list and, for every item that isn't in the second list but is in the first list, add it to the third list. E.g:

temp3 = []
for i in temp1:
    if i not in temp2:
        temp3.append(i)
print(temp3)

Do you recommend using semicolons after every statement in JavaScript?

JavaScript automatically inserts semicolons whilst interpreting your code, so if you put the value of the return statement below the line, it won't be returned:

Your Code:

return
5

JavaScript Interpretation:

return;
5;

Thus, nothing is returned, because of JavaScript's auto semicolon insertion

Correct syntax to compare values in JSTL <c:if test="${values.type}=='object'">

The comparison needs to be evaluated fully inside EL ${ ... }, not outside.

<c:if test="${values.type eq 'object'}">

As to the docs, those ${} things are not JSTL, but EL (Expression Language) which is a whole subject at its own. JSTL (as every other JSP taglib) is just utilizing it. You can find some more EL examples here.

<c:if test="#{bean.booleanValue}" />
<c:if test="#{bean.intValue gt 10}" />
<c:if test="#{bean.objectValue eq null}" />
<c:if test="#{bean.stringValue ne 'someValue'}" />
<c:if test="#{not empty bean.collectionValue}" />
<c:if test="#{not bean.booleanValue and bean.intValue ne 0}" />
<c:if test="#{bean.enumValue eq 'ONE' or bean.enumValue eq 'TWO'}" />

See also:


By the way, unrelated to the concrete problem, if I guess your intent right, you could also just call Object#getClass() and then Class#getSimpleName() instead of adding a custom getter.

<c:forEach items="${list}" var="value">
    <c:if test="${value['class'].simpleName eq 'Object'}">
        <!-- code here -->
    </c:if>
</c:forEeach>

See also:

const vs constexpr on variables

I believe there is a difference. Let's rename them so that we can talk about them more easily:

const     double PI1 = 3.141592653589793;
constexpr double PI2 = 3.141592653589793;

Both PI1 and PI2 are constant, meaning you can not modify them. However only PI2 is a compile-time constant. It shall be initialized at compile time. PI1 may be initialized at compile time or run time. Furthermore, only PI2 can be used in a context that requires a compile-time constant. For example:

constexpr double PI3 = PI1;  // error

but:

constexpr double PI3 = PI2;  // ok

and:

static_assert(PI1 == 3.141592653589793, "");  // error

but:

static_assert(PI2 == 3.141592653589793, "");  // ok

As to which you should use? Use whichever meets your needs. Do you want to ensure that you have a compile time constant that can be used in contexts where a compile-time constant is required? Do you want to be able to initialize it with a computation done at run time? Etc.

Use of PUT vs PATCH methods in REST API real life scenarios

Everyone else has answered the PUT vs PATCH. I was just going to answer what part of the title of the original question asks: "... in REST API real life scenarios". In the real world, this happened to me with internet application that had a RESTful server and a relational database with a Customer table that was "wide" (about 40 columns). I mistakenly used PUT but had assumed it was like a SQL Update command and had not filled out all the columns. Problems: 1) Some columns were optional (so blank was valid answer), 2) many columns rarely changed, 3) some columns the user was not allowed to change such as time stamp of Last Purchase Date, 4) one column was a free-form text "Comments" column that users diligently filled with half-page customer services comments like spouses name to ask about OR usual order, 5) I was working on an internet app at time and there was worry about packet size.

The disadvantage of PUT is that it forces you to send a large packet of info (all columns including the entire Comments column, even though only a few things changed) AND multi-user issue of 2+ users editing the same customer simultaneously (so last one to press Update wins). The disadvantage of PATCH is that you have to keep track on the view/screen side of what changed and have some intelligence to send only the parts that changed. Patch's multi-user issue is limited to editing the same column(s) of same customer.

How to scanf only integer and repeat reading if the user enters non-numeric characters?

#include <stdio.h>
main()
{
    char str[100];
    int num;
    while(1) {
        printf("Enter a number: ");
        scanf("%[^0-9]%d",str,&num);
        printf("You entered the number %d\n",num);
    }
    return 0;
}

%[^0-9] in scanf() gobbles up all that is not between 0 and 9. Basically it cleans the input stream of non-digits and puts it in str. Well, the length of non-digit sequence is limited to 100. The following %d selects only integers in the input stream and places it in num.

Get local IP address

Dns.GetHostEntry(Dns.GetHostName()).AddressList[1].MapToIPv4() //returns 192.168.14.1

enter image description here

How to format number of decimal places in wpf using style/template?

    void NumericTextBoxInput(object sender, TextCompositionEventArgs e)
    {
        TextBox txt = (TextBox)sender;
        var regex = new Regex(@"^[0-9]*(?:\.[0-9]{0,1})?$");
        string str = txt.Text + e.Text.ToString();
        int cntPrc = 0;
        if (str.Contains('.'))
        {
            string[] tokens = str.Split('.');
            if (tokens.Count() > 0)
            {
                string result = tokens[1];
                char[] prc = result.ToCharArray();
                cntPrc = prc.Count();
            }
        }
        if (regex.IsMatch(e.Text) && !(e.Text == "." && ((TextBox)sender).Text.Contains(e.Text)) && (cntPrc < 3))
        {
            e.Handled = false;
        }
        else
        {
            e.Handled = true;
        }
    }

How to set NODE_ENV to production/development in OS X

It might be a chance that you have made two instances of sequelize object

for example : var con1=new Sequelize(); var con2=new Sequelize();

than also same error will occur

Facebook Graph API v2.0+ - /me/friends returns empty, or only friends who also use my application

As Simon mentioned, this is not possible in the new Facebook API. Pure technically speaking you can do it via browser automation.

  • this is against Facebook policy, so depending on the country where you live, this may not be legal
  • you'll have to use your credentials / ask user for credentials and possibly store them (storing passwords even symmetrically encrypted is not a good idea)
  • when Facebook changes their API, you'll have to update the browser automation code as well (if you can't force updates of your application, you should put browser automation piece out as a webservice)
  • this is bypassing the OAuth concept
  • on the other hand, my feeling is that I'm owning my data including the list of my friends and Facebook shouldn't restrict me from accessing those via the API

Sample implementation using WatiN:

class FacebookUser
{
  public string Name { get; set; }
  public long Id { get; set; }
}

public IList<FacebookUser> GetFacebookFriends(string email, string password, int? maxTimeoutInMilliseconds)
{
  var users = new List<FacebookUser>();
  Settings.Instance.MakeNewIeInstanceVisible = false;
  using (var browser = new IE("https://www.facebook.com"))
  {
    try
    {
      browser.TextField(Find.ByName("email")).Value = email;
      browser.TextField(Find.ByName("pass")).Value = password;
      browser.Form(Find.ById("login_form")).Submit();
      browser.WaitForComplete();
    }
    catch (ElementNotFoundException)
    {
      // We're already logged in
    }
    browser.GoTo("https://www.facebook.com/friends");
    var watch = new Stopwatch();
    watch.Start();

    Link previousLastLink = null;
    while (maxTimeoutInMilliseconds.HasValue && watch.Elapsed.TotalMilliseconds < maxTimeoutInMilliseconds.Value)
    {
      var lastLink = browser.Links.Where(l => l.GetAttributeValue("data-hovercard") != null
                       && l.GetAttributeValue("data-hovercard").Contains("user.php")
                       && l.Text != null
                     ).LastOrDefault();

      if (lastLink == null || previousLastLink == lastLink)
      {
        break;
      }

      var ieElement = lastLink.NativeElement as IEElement;
      if (ieElement != null)
      {
        var htmlElement = ieElement.AsHtmlElement;
        htmlElement.scrollIntoView();
        browser.WaitForComplete();
      }

      previousLastLink = lastLink;
    }

    var links = browser.Links.Where(l => l.GetAttributeValue("data-hovercard") != null
      && l.GetAttributeValue("data-hovercard").Contains("user.php")
      && l.Text != null
    ).ToList();

    var idRegex = new Regex("id=(?<id>([0-9]+))");
    foreach (var link in links)
    {
      string hovercard = link.GetAttributeValue("data-hovercard");
      var match = idRegex.Match(hovercard);
      long id = 0;
      if (match.Success)
      {
        id = long.Parse(match.Groups["id"].Value);
      }
      users.Add(new FacebookUser
      {
        Name = link.Text,
        Id = id
      });
    }
  }
  return users;
}

Prototype with implementation of this approach (using C#/WatiN) see https://github.com/svejdo1/ShadowApi. It is also allowing dynamic update of Facebook connector that is retrieving a list of your contacts.

How to destroy a JavaScript object?

You can't delete objects, they are removed when there are no more references to them. You can delete references with delete.

However, if you have created circular references in your objects you may have to de-couple some things.

Stripping non printable characters from a string in python

The one below performs faster than the others above. Take a look

''.join([x if x in string.printable else '' for x in Str])

What's the quickest way to multiply multiple cells by another number?

As one of the answers above says: " then drag the formula fill handle." This KEY feature is not mentioned in MS's explanation, nor in others here. I spent over an hour trying to follow the various instructions, to no avail. This is because you have to click and hold near the bottom of the cell just right (and at least on my computer that is not at all easy) so that a sort of "handle" appears. Once you're luck enough to get that, then carefully slide ["drag"] your cursor down to the lowermost of the cells you want to be multiplied by the constant. The products should show up in each cell as you move down. Just dragging down will give you only the answer in the first cell and a lot of white space.

Merge 2 arrays of objects

Merging two arrays:

var arr1 = new Array({name: "lang", value: "English"}, {name: "age", value: "18"});
var arr2 = new Array({name : "childs", value: '5'}, {name: "lang", value: "German"});
var result=arr1.concat(arr2);
// result: [{name: "lang", value: "English"}, {name: "age", value: "18"}, {name : "childs", value: '5'}, {name: "lang", value: "German"}]

Merging two arrays without duplicated values for 'name':

var arr1 = new Array({name: "lang", value: "English"}, {name: "age", value: "18"});
var arr2 = new Array({name : "childs", value: '5'}, {name: "lang", value: "German"});
var i,p,obj={},result=[];
for(i=0;i<arr1.length;i++)obj[arr1[i].name]=arr1[i].value;
for(i=0;i<arr2.length;i++)obj[arr2[i].name]=arr2[i].value;
for(p in obj)if(obj.hasOwnProperty(p))result.push({name:p,value:obj[p]});
// result: [{name: "lang", value: "German"}, {name: "age", value: "18"}, {name : "childs", value: '5'}]

Using os.walk() to recursively traverse directories in Python

Would be the best way

def traverse_dir_recur(dir):
    import os
    l = os.listdir(dir)
    for d in l:
        if os.path.isdir(dir + d):
            traverse_dir_recur(dir+  d +"/")
        else:
            print(dir + d)

How to write multiple line string using Bash with variables?

Below mechanism helps in redirecting multiple lines to file. Keep complete string under " so that we can redirect values of the variable.

#!/bin/bash
kernel="2.6.39"
echo "line 1, ${kernel}
line 2," > a.txt
echo 'line 2, ${kernel}
line 2,' > b.txt

Content of a.txt is

line 1, 2.6.39
line 2,

Content of b.txt is

line 2, ${kernel}
line 2,

Upgrade python in a virtualenv

Step 1: Freeze requirement & take a back-up of existing env

pip freeze > requirements.txt
deactivate
mv env env_old

Step 2: Install Python 3.7 & activate virutal environment

sudo apt-get install python3.7-venv
python3.7 -m venv env
source env/bin/activate
python --version

Step 3: Install requirements

sudo apt-get install python3.7-dev
pip3 install -r requirements.txt

write a shell script to ssh to a remote machine and execute commands

There are a number of ways to handle this.

My favorite way is to install http://pamsshagentauth.sourceforge.net/ on the remote systems and also your own public key. (Figure out a way to get these installed on the VM, somehow you got an entire Unix system installed, what's a couple more files?)

With your ssh agent forwarded, you can now log in to every system without a password.

And even better, that pam module will authenticate for sudo with your ssh key pair so you can run with root (or any other user's) rights as needed.

You don't need to worry about the host key interaction. If the input is not a terminal then ssh will just limit your ability to forward agents and authenticate with passwords.

You should also look into packages like Capistrano. Definitely look around that site; it has an introduction to remote scripting.

Individual script lines might look something like this:

ssh remote-system-name command arguments ... # so, for exmaple,
ssh target.mycorp.net sudo puppet apply

Converting a char to ASCII?

Uhm, what's wrong with this:

#include <iostream>

using namespace std;

int main(int, char **)
{
    char c = 'A';

    int x = c; // Look ma! No cast!

    cout << "The character '" << c << "' has an ASCII code of " << x << endl;

    return 0;
}

Are there any SHA-256 javascript implementations that are generally considered trustworthy?

On https://developer.mozilla.org/en-US/docs/Web/API/SubtleCrypto/digest I found this snippet that uses internal js module:

async function sha256(message) {
    // encode as UTF-8
    const msgBuffer = new TextEncoder().encode(message);                    

    // hash the message
    const hashBuffer = await crypto.subtle.digest('SHA-256', msgBuffer);

    // convert ArrayBuffer to Array
    const hashArray = Array.from(new Uint8Array(hashBuffer));

    // convert bytes to hex string                  
    const hashHex = hashArray.map(b => ('00' + b.toString(16)).slice(-2)).join('');
    return hashHex;
}

Note that crypto.subtle in only available on https or localhost - for example for your local development with python3 -m http.server you need to add this line to your /etc/hosts: 0.0.0.0 localhost

Reboot - and you can open localhost:8000 with working crypto.subtle.

Select multiple images from android gallery

     // for choosing multiple images declare variables
     int PICK_IMAGE_MULTIPLE = 2;
     String realImagePath;

     // After requesting FILE READ PERMISSION may be on button click
     Intent intent = new Intent();
     intent.setType("image/*");
     intent.putExtra(Intent.EXTRA_ALLOW_MULTIPLE, true);
     intent.setAction(Intent.ACTION_GET_CONTENT);
     startActivityForResult(Intent.createChooser(intent,"Select Images"), PICK_IMAGE_MULTIPLE);


    public void onActivityResult(int requestCode, int resultCode, Intent data) {
        super.onActivityResult(requestCode, resultCode, data);// FOR CHOOSING MULTIPLE IMAGES
        try {
            // When an Image is picked
            if (requestCode == PICK_IMAGE_MULTIPLE && resultCode == RESULT_OK
                    && null != data) {
                if (data.getClipData() != null) {
                    int count = data.getClipData().getItemCount(); //evaluate the count before the for loop --- otherwise, the count is evaluated every loop.
                    for (int i = 0; i < count; i++) {
                        Uri imageUri = data.getClipData().getItemAt(i).getUri();
                        realImagePath = getPath(this, imageUri);
                        //do something with the image (save it to some directory or whatever you need to do with it here)
                        Log.e("ImagePath", "onActivityResult: " + realImagePath);
                    }
                } else if (data.getData() != null) {
                    Uri imageUri = data.getData();
                    realImagePath = getPath(this, imageUri);
                    //do something with the image (save it to some directory or whatever you need to do with it here)
                    Log.e("ImagePath", "onActivityResult: " + realImagePath);
                }
            }
        } catch (Exception e) {
            Toast.makeText(this, "Something went wrong", Toast.LENGTH_LONG)
                    .show();
        }
    }
     

    public static String getPath(final Context context, final Uri uri) {
        // DocumentProvider
        if (DocumentsContract.isDocumentUri(context, uri)) {
            // ExternalStorageProvider
            if (isExternalStorageDocument(uri)) {
                final String docId = DocumentsContract.getDocumentId(uri);
                final String[] split = docId.split(":");
                final String type = split[0];
    
                if ("primary".equalsIgnoreCase(type)) {
                    return Environment.getExternalStorageDirectory() + "/" + split[1];
                }
    
                // TODO handle non-primary volumes
            }
            // DownloadsProvider
            else if (isDownloadsDocument(uri)) {
    
                final String id = DocumentsContract.getDocumentId(uri);
                final Uri contentUri = ContentUris.withAppendedId(
                        Uri.parse("content://downloads/public_downloads"), Long.parseLong(id));
    
                return getDataColumn(context, contentUri, null, null);
            }
            // MediaProvider
            else if (isMediaDocument(uri)) {
                final String docId = DocumentsContract.getDocumentId(uri);
                final String[] split = docId.split(":");
                final String type = split[0];
    
                Uri contentUri = null;
                if ("image".equals(type)) {
                    contentUri = MediaStore.Images.Media.EXTERNAL_CONTENT_URI;
                } else if ("video".equals(type)) {
                    contentUri = MediaStore.Video.Media.EXTERNAL_CONTENT_URI;
                } else if ("audio".equals(type)) {
                    contentUri = MediaStore.Audio.Media.EXTERNAL_CONTENT_URI;
                }
    
                final String selection = "_id=?";
                final String[] selectionArgs = new String[]{
                        split[1]
                };
    
                return getDataColumn(context, contentUri, selection, selectionArgs);
            }
        }
        // MediaStore (and general)
        else if ("content".equalsIgnoreCase(uri.getScheme())) {
            return getDataColumn(context, uri, null, null);
        }
        // File
        else if ("file".equalsIgnoreCase(uri.getScheme())) {
            return uri.getPath();
        }
    
        return null;
    }
    
    /**
     * Get the value of the data column for this Uri. This is useful for
     * MediaStore Uris, and other file-based ContentProviders.
     *
     * @param context       The context.
     * @param uri           The Uri to query.
     * @param selection     (Optional) Filter used in the query.
     * @param selectionArgs (Optional) Selection arguments used in the query.
     * @return The value of the _data column, which is typically a file path.
     */
    public static String getDataColumn(Context context, Uri uri, String selection,
                                       String[] selectionArgs) {
    
        Cursor cursor = null;
        final String column = "_data";
        final String[] projection = {
                column
        };
    
        try {
            cursor = context.getContentResolver().query(uri, projection, selection, selectionArgs,
                    null);
            if (cursor != null && cursor.moveToFirst()) {
                final int column_index = cursor.getColumnIndexOrThrow(column);
                return cursor.getString(column_index);
            }
        } finally {
            if (cursor != null)
                cursor.close();
        }
        return null;
    }
    
    
    /**
     * @param uri The Uri to check.
     * @return Whether the Uri authority is ExternalStorageProvider.
     */
    public static boolean isExternalStorageDocument(Uri uri) {
        return "com.android.externalstorage.documents".equals(uri.getAuthority());
    }
    
    /**
     * @param uri The Uri to check.
     * @return Whether the Uri authority is DownloadsProvider.
     */
    public static boolean isDownloadsDocument(Uri uri) {
        return "com.android.providers.downloads.documents".equals(uri.getAuthority());
    }
    
    /**
     * @param uri The Uri to check.
     * @return Whether the Uri authority is MediaProvider.
     */
    public static boolean isMediaDocument(Uri uri) {
        return "com.android.providers.media.documents".equals(uri.getAuthority());
    }

this worked perfectly for me credits: Get real path from URI, Android KitKat new storage access framework

How to get absolute path to file in /resources folder of your project

There are two problems on our way to the absolute path:

  1. The placement found will be not where the source files lie, but where the class is saved. And the resource folder almost surely will lie somewhere in the source folder of the project.
  2. The same functions for retrieving the resource work differently if the class runs in a plugin or in a package directly in the workspace.

The following code will give us all useful paths:

    URL localPackage = this.getClass().getResource("");
    URL urlLoader = YourClassName.class.getProtectionDomain().getCodeSource().getLocation();
    String localDir = localPackage.getPath();
    String loaderDir = urlLoader.getPath();
    System.out.printf("loaderDir = %s\n localDir = %s\n", loaderDir, localDir);

Here both functions that can be used for localization of the resource folder are researched. As for class, it can be got in either way, statically or dynamically.


If the project is not in the plugin, the code if run in JUnit, will print:

loaderDir = /C:.../ws/source.dir/target/test-classes/
 localDir = /C:.../ws/source.dir/target/test-classes/package/

So, to get to src/rest/resources we should go up and down the file tree. Both methods can be used. Notice, we can't use getResource(resourceFolderName), for that folder is not in the target folder. Nobody puts resources in the created folders, I hope.


If the class is in the package that is in the plugin, the output of the same test will be:

loaderDir = /C:.../ws/plugin/bin/
 localDir = /C:.../ws/plugin/bin/package/

So, again we should go up and down the folder tree.


The most interesting is the case when the package is launched in the plugin. As JUnit plugin test, for our example. The output is:

loaderDir = /C:.../ws/plugin/
 localDir = /package/

Here we can get the absolute path only combining the results of both functions. And it is not enough. Between them we should put the local path of the place where the classes packages are, relatively to the plugin folder. Probably, you will have to insert something as src or src/test/resource here.

You can insert the code into yours and see the paths that you have.

qmake: could not find a Qt installation of ''

sudo apt-get install qt5-default works for me.

$ aptitude show qt5-default
tells that

This package sets Qt 5 to be the default Qt version to be used when using development binaries like qmake. It provides a default configuration for qtchooser, but does not prevent alternative Qt installations from being used.

What does `dword ptr` mean?

The dword ptr part is called a size directive. This page explains them, but it wasn't possible to direct-link to the correct section.

Basically, it means "the size of the target operand is 32 bits", so this will bitwise-AND the 32-bit value at the address computed by taking the contents of the ebp register and subtracting four with 0.

What is a "web service" in plain English?

Web services are almost like normal a web page. The difference is that they are formatted to make it very easy for a program to pull data from the page, to the point of probably not using any HTML. They generally also are more reliable as to the consistency of the format, may use a different formal process to define the content such soap or raw xml, and there is often also a descriptor document that formally defines the structure for the data.

If you can decode JWT, how are they secure?

Only JWT's privateKey, which is on your server will decrypt the encrypted JWT. Those who know the privateKey will be able to decrypt the encrypted JWT.

Hide the privateKey in a secure location in your server and never tell anyone the privateKey.

Where to download Microsoft Visual c++ 2003 redistributable

After a bit of googling, it seems that there never was a separate redistributable for Visual C++ 2003 (7.1). At least that is what a post on the microsoft forum says.

You may however be able to extract the runtime DLLs from the VC 7.1 DST timezone update.

How do I connect to a SQL Server 2008 database using JDBC?

You can use this :

import java.sql.Connection;
import java.sql.DriverManager;
import java.sql.ResultSet;
import java.sql.Statement;

public class ConnectMSSQLServer
{
   public void dbConnect(String db_connect_string,
            String db_userid,
            String db_password)
   {
      try {
         Class.forName("com.microsoft.sqlserver.jdbc.SQLServerDriver");
         Connection conn = DriverManager.getConnection(db_connect_string,
                  db_userid, db_password);
         System.out.println("connected");
         Statement statement = conn.createStatement();
         String queryString = "select * from sysobjects where type='u'";
         ResultSet rs = statement.executeQuery(queryString);
         while (rs.next()) {
            System.out.println(rs.getString(1));
         }
      } catch (Exception e) {
         e.printStackTrace();
      }
   }

   public static void main(String[] args)
   {
      ConnectMSSQLServer connServer = new ConnectMSSQLServer();
      connServer.dbConnect("jdbc:sqlserver://<hostname>", "<user>",
               "<password>");
   }
}

c#: getter/setter

With the release of C# 6, you can now do something like this for private properties.

public constructor()
{
   myProp = "some value";
}

public string myProp { get; }

How to add text inside the doughnut chart using Chart.js?

This is based on Cmyker's update for Chart.js 2. (posted as another answer as I can't comment yet)

I had an issue with the text alignment on Chrome when the legend is displayed as the chart height does not include this so it's not aligned correctly in the middle. Fixed this by accounting for this in the calculation of fontSize and textY.

I calculated percentage inside the method rather than a set value as I have multiple of these on the page. Assumptions are that your chart only has 2 values (otherwise what is the percentage of? and that the first is the one you want to show the percentage for. I have a bunch of other charts too so I do a check for type = doughnut. I'm only using doughnuts to show percentages so it works for me.

Text color seems a bit hit and miss depending on what order things run in etc so I ran into an issue when resizing that the text would change color (between black and the primary color in one case, and secondary color and white in another) so I "save" whatever the existing fill style was, draw the text (in the color of the primary data) then restore the old fill style. (Preserving the old fill style doesn't seem needed but you never know.)

https://jsfiddle.net/g733tj8h/

Chart.pluginService.register({
  beforeDraw: function(chart) {
    var width = chart.chart.width,
        height = chart.chart.height,
        ctx = chart.chart.ctx,
        type = chart.config.type;

    if (type == 'doughnut')
    {
      var percent = Math.round((chart.config.data.datasets[0].data[0] * 100) /
                    (chart.config.data.datasets[0].data[0] +
                    chart.config.data.datasets[0].data[1]));
      var oldFill = ctx.fillStyle;
      var fontSize = ((height - chart.chartArea.top) / 100).toFixed(2);

      ctx.restore();
      ctx.font = fontSize + "em sans-serif";
      ctx.textBaseline = "middle"

      var text = percent + "%",
          textX = Math.round((width - ctx.measureText(text).width) / 2),
          textY = (height + chart.chartArea.top) / 2;

      ctx.fillStyle = chart.config.data.datasets[0].backgroundColor[0];
      ctx.fillText(text, textX, textY);
      ctx.fillStyle = oldFill;
      ctx.save();
    }
  }
});

_x000D_
_x000D_
var data = {_x000D_
  labels: ["Red","Blue"],_x000D_
  datasets: [_x000D_
    {_x000D_
      data: [300, 50],_x000D_
      backgroundColor: ["#FF6384","#36A2EB"],_x000D_
    }]_x000D_
};_x000D_
_x000D_
Chart.pluginService.register({_x000D_
  beforeDraw: function(chart) {_x000D_
    var width = chart.chart.width,_x000D_
        height = chart.chart.height,_x000D_
        ctx = chart.chart.ctx,_x000D_
        type = chart.config.type;_x000D_
_x000D_
    if (type == 'doughnut')_x000D_
    {_x000D_
     var percent = Math.round((chart.config.data.datasets[0].data[0] * 100) /_x000D_
                    (chart.config.data.datasets[0].data[0] +_x000D_
                    chart.config.data.datasets[0].data[1]));_x000D_
   var oldFill = ctx.fillStyle;_x000D_
      var fontSize = ((height - chart.chartArea.top) / 100).toFixed(2);_x000D_
      _x000D_
      ctx.restore();_x000D_
      ctx.font = fontSize + "em sans-serif";_x000D_
      ctx.textBaseline = "middle"_x000D_
_x000D_
      var text = percent + "%",_x000D_
          textX = Math.round((width - ctx.measureText(text).width) / 2),_x000D_
          textY = (height + chart.chartArea.top) / 2;_x000D_
   _x000D_
      ctx.fillStyle = chart.config.data.datasets[0].backgroundColor[0];_x000D_
      ctx.fillText(text, textX, textY);_x000D_
      ctx.fillStyle = oldFill;_x000D_
      ctx.save();_x000D_
    }_x000D_
  }_x000D_
});_x000D_
_x000D_
var myChart = new Chart(document.getElementById('myChart'), {_x000D_
  type: 'doughnut',_x000D_
  data: data,_x000D_
  options: {_x000D_
   responsive: true,_x000D_
    legend: {_x000D_
      display: true_x000D_
    }_x000D_
  }_x000D_
});
_x000D_
<script src="https://cdnjs.cloudflare.com/ajax/libs/Chart.js/2.1.6/Chart.bundle.js"></script>_x000D_
<canvas id="myChart"></canvas>
_x000D_
_x000D_
_x000D_

Regex pattern including all special characters

Try using this for the same things - StringUtils.isAlphanumeric(value)

PLS-00428: an INTO clause is expected in this SELECT statement

In PLSQL block, columns of select statements must be assigned to variables, which is not the case in SQL statements.

The second BEGIN's SQL statement doesn't have INTO clause and that caused the error.

DECLARE
   PROD_ROW_ID   VARCHAR (10) := NULL;
   VIS_ROW_ID    NUMBER;
   DSC           VARCHAR (512);
BEGIN
   SELECT ROW_ID
     INTO VIS_ROW_ID
     FROM SIEBEL.S_PROD_INT
    WHERE PART_NUM = 'S0146404';

   BEGIN
      SELECT    RTRIM (VIS.SERIAL_NUM)
             || ','
             || RTRIM (PLANID.DESC_TEXT)
             || ','
             || CASE
                   WHEN PLANID.HIGH = 'TEST123'
                   THEN
                      CASE
                         WHEN TO_DATE (PROD.START_DATE) + 30 > SYSDATE
                         THEN
                            'Y'
                         ELSE
                            'N'
                      END
                   ELSE
                      'N'
                END
             || ','
             || 'GB'
             || ','
             || RTRIM (TO_CHAR (PROD.START_DATE, 'YYYY-MM-DD'))
        INTO DSC
        FROM SIEBEL.S_LST_OF_VAL PLANID
             INNER JOIN SIEBEL.S_PROD_INT PROD
                ON PROD.PART_NUM = PLANID.VAL
             INNER JOIN SIEBEL.S_ASSET NETFLIX
                ON PROD.PROD_ID = PROD.ROW_ID
             INNER JOIN SIEBEL.S_ASSET VIS
                ON VIS.PROM_INTEG_ID = PROD.PROM_INTEG_ID
             INNER JOIN SIEBEL.S_PROD_INT VISPROD
                ON VIS.PROD_ID = VISPROD.ROW_ID
       WHERE     PLANID.TYPE = 'Test Plan'
             AND PLANID.ACTIVE_FLG = 'Y'
             AND VISPROD.PART_NUM = VIS_ROW_ID
             AND PROD.STATUS_CD = 'Active'
             AND VIS.SERIAL_NUM IS NOT NULL;
   END;
END;
/

References

http://docs.oracle.com/cd/E11882_01/appdev.112/e25519/static.htm#LNPLS00601 http://docs.oracle.com/cd/B19306_01/appdev.102/b14261/selectinto_statement.htm#CJAJAAIG http://pls-00428.ora-code.com/

NSArray + remove item from array

As others suggested, NSMutableArray has methods to do so but sometimes you are forced to use NSArray, I'd use:

NSArray* newArray = [oldArray subarrayWithRange:NSMakeRange(1, [oldArray count] - 1)];

This way, the oldArray stays as it was but a newArray will be created with the first item removed.

How can I count the rows with data in an Excel sheet?

You should use the sumif function in Excel:

=SUMIF(A5:C10;"Text_to_find";C5:C10)

This function takes a range like this square A5:C10 then you have some text to find this text can be in A or B then it will add the number from the C-row.

Pandas: Appending a row to a dataframe and specify its index label

The name of the Series becomes the index of the row in the DataFrame:

In [99]: df = pd.DataFrame(np.random.randn(8, 4), columns=['A','B','C','D'])

In [100]: s = df.xs(3)

In [101]: s.name = 10

In [102]: df.append(s)
Out[102]: 
           A         B         C         D
0  -2.083321 -0.153749  0.174436  1.081056
1  -1.026692  1.495850 -0.025245 -0.171046
2   0.072272  1.218376  1.433281  0.747815
3  -0.940552  0.853073 -0.134842 -0.277135
4   0.478302 -0.599752 -0.080577  0.468618
5   2.609004 -1.679299 -1.593016  1.172298
6  -0.201605  0.406925  1.983177  0.012030
7   1.158530 -2.240124  0.851323 -0.240378
10 -0.940552  0.853073 -0.134842 -0.277135

What is difference between @RequestBody and @RequestParam?

Here is an example with @RequestBody, First look at the controller !!

  public ResponseEntity<Void> postNewProductDto(@RequestBody NewProductDto newProductDto) {

   ...
        productService.registerProductDto(newProductDto);
        return new ResponseEntity<>(HttpStatus.CREATED);
   ....

}

And here is angular controller

function postNewProductDto() {
                var url = "/admin/products/newItem";
                $http.post(url, vm.newProductDto).then(function () {
                            //other things go here...
                            vm.newProductMessage = "Product successful registered";
                        }
                        ,
                        function (errResponse) {
                            //handling errors ....
                        }
                );
            }

And a short look at form

 <label>Name: </label>
 <input ng-model="vm.newProductDto.name" />

<label>Price </label> 
 <input ng-model="vm.newProductDto.price"/>

 <label>Quantity </label>
  <input ng-model="vm.newProductDto.quantity"/>

 <label>Image </label>
 <input ng-model="vm.newProductDto.photo"/>

 <Button ng-click="vm.postNewProductDto()" >Insert Item</Button>

 <label > {{vm.newProductMessage}} </label>

Are there inline functions in java?

No, there is no inline function in java. Yes, you can use a public static method anywhere in the code when placed in a public class. The java compiler may do inline expansion on a static or final method, but that is not guaranteed.

Typically such code optimizations are done by the compiler in combination with the JVM/JIT/HotSpot for code segments used very often. Also other optimization concepts like register declaration of parameters are not known in java.

Optimizations cannot be forced by declaration in java, but done by compiler and JIT. In many other languages these declarations are often only compiler hints (you can declare more register parameters than the processor has, the rest is ignored).

Declaring java methods static, final or private are also hints for the compiler. You should use it, but no garantees. Java performance is dynamic, not static. First call to a system is always slow because of class loading. Next calls are faster, but depending on memory and runtime the most common calls are optimized withinthe running system, so a server may become faster during runtime!

How to sum the values of a JavaScript object?

You could put it all in one function:

_x000D_
_x000D_
function sum( obj ) {_x000D_
  var sum = 0;_x000D_
  for( var el in obj ) {_x000D_
    if( obj.hasOwnProperty( el ) ) {_x000D_
      sum += parseFloat( obj[el] );_x000D_
    }_x000D_
  }_x000D_
  return sum;_x000D_
}_x000D_
    _x000D_
var sample = { a: 1 , b: 2 , c:3 };_x000D_
var summed = sum( sample );_x000D_
console.log( "sum: "+summed );
_x000D_
_x000D_
_x000D_


For fun's sake here is another implementation using Object.keys() and Array.reduce() (browser support should not be a big issue anymore):

_x000D_
_x000D_
function sum(obj) {_x000D_
  return Object.keys(obj).reduce((sum,key)=>sum+parseFloat(obj[key]||0),0);_x000D_
}_x000D_
let sample = { a: 1 , b: 2 , c:3 };_x000D_
_x000D_
console.log(`sum:${sum(sample)}`);
_x000D_
_x000D_
_x000D_

But this seems to be way slower: jsperf.com

Updating the value of data attribute using jQuery

$('.toggle img').data('block', 'something').attr('src', 'something.jpg');

How to get complete current url for Cakephp

I use $this->here for the path, to get the whole URL you'll have to do as Juhana said and use the $_SERVER variables. There's no need to use a Cake function for this.

The backend version is not supported to design database diagrams or tables

You only get that message if you try to use Designer or diagrams. If you use t-SQL it works fine:

Select * 

into newdb.dbo.newtable
from olddb.dbo.yourtable

where olddb.dbo.yourtable has been created in 2008 exactly as you want the table to be in 2012

What is the best open-source java charting library? (other than jfreechart)

There is JChart which is all open source. I'm not sure exactly what you are graphing and how you are graphing it (servlets, swing, etc) so I would say just look at a couple different ones and see which works for you.

http://sourceforge.net/projects/jchart/

I've also used JGraph but I've only used their commercial version. They do offer an open source version however:

http://www.jgraph.com/jgraph.html

How to stop (and restart) the Rails Server?

Press Ctrl+C

When you start the server it mentions this in the startup text.

how to add value to a tuple?

As other people have answered, tuples in python are immutable and the only way to 'modify' one is to create a new one with the appended elements included.

But the best solution is a list. When whatever function or method that requires a tuple needs to be called, create a tuple by using tuple(list).

Copy table without copying data

Try

CREATE TABLE foo LIKE bar;

so the keys and indexes are copied over as, well.

Documentation

The type or namespace name does not exist in the namespace 'System.Web.Mvc'

I have a project that does this whenever I build with the View open. As soon as I closed the view, the error goes away and the build succeeds. Very strange.

Can table columns with a Foreign Key be NULL?

I found that when inserting, the null column values had to be specifically declared as NULL, otherwise I would get a constraint violation error (as opposed to an empty string).

How might I convert a double to the nearest integer value?

Methods in other answers throw OverflowException if the float value is outside the Int range. https://docs.microsoft.com/en-us/dotnet/api/system.convert.toint32?view=netframework-4.8#System_Convert_ToInt32_System_Single_

int result = 0;
try {
    result = Convert.ToInt32(value);
}
catch (OverflowException) {
    if (value > 0) result = int.MaxValue;
    else result = int.Minvalue;
}

"Error: Main method not found in class MyClass, please define the main method as..."

The problem is that you do not have a public void main(String[] args) method in the class you attempt to invoke.

It

  • must be static
  • must have exactly one String array argument (which may be named anything)
  • must be spelled m-a-i-n in lowercase.

Note, that you HAVE actually specified an existing class (otherwise the error would have been different), but that class lacks the main method.

Log record changes in SQL server in an audit table

I know this is old, but maybe this will help someone else.

Do not log "new" values. Your existing table, GUESTS, has the new values. You'll have double entry of data, plus your DB size will grow way too fast that way.

I cleaned this up and minimized it for this example, but here is the tables you'd need for logging off changes:

CREATE TABLE GUESTS (
      GuestID INT IDENTITY(1,1) PRIMARY KEY, 
      GuestName VARCHAR(50), 
      ModifiedBy INT, 
      ModifiedOn DATETIME
)

CREATE TABLE GUESTS_LOG (
      GuestLogID INT IDENTITY(1,1) PRIMARY KEY, 
      GuestID INT, 
      GuestName VARCHAR(50), 
      ModifiedBy INT, 
      ModifiedOn DATETIME
)

When a value changes in the GUESTS table (ex: Guest name), simply log off that entire row of data, as-is, to your Log/Audit table using the Trigger. Your GUESTS table has current data, the Log/Audit table has the old data.

Then use a select statement to get data from both tables:

SELECT 0 AS 'GuestLogID', GuestID, GuestName, ModifiedBy, ModifiedOn FROM [GUESTS] WHERE GuestID = 1
UNION
SELECT GuestLogID, GuestID, GuestName, ModifiedBy, ModifiedOn FROM [GUESTS_LOG] WHERE GuestID = 1
ORDER BY ModifiedOn ASC

Your data will come out with what the table looked like, from Oldest to Newest, with the first row being what was created & the last row being the current data. You can see exactly what changed, who changed it, and when they changed it.

Optionally, I used to have a function that looped through the RecordSet (in Classic ASP), and only displayed what values had changed on the web page. It made for a GREAT audit trail so that users could see what had changed over time.

Conditional Formatting using Excel VBA code

I think I just discovered a way to apply overlapping conditions in the expected way using VBA. After hours of trying out different approaches I found that what worked was changing the "Applies to" range for the conditional format rule, after every single one was created!

This is my working example:

Sub ResetFormatting()
' ----------------------------------------------------------------------------------------
' Written by..: Julius Getz Mørk
' Purpose.....: If conditional formatting ranges are broken it might cause a huge increase
'               in duplicated formatting rules that in turn will significantly slow down
'               the spreadsheet.
'               This macro is designed to reset all formatting rules to default.
' ---------------------------------------------------------------------------------------- 

On Error GoTo ErrHandler

' Make sure we are positioned in the correct sheet
WS_PROMO.Select

' Disable Events
Application.EnableEvents = False

' Delete all conditional formatting rules in sheet
Cells.FormatConditions.Delete

' CREATE ALL THE CONDITIONAL FORMATTING RULES:

' (1) Make negative values red
With Cells(1, 1).FormatConditions.add(xlCellValue, xlLess, "=0")
    .Font.Color = -16776961
    .StopIfTrue = False
End With

' (2) Highlight defined good margin as green values
With Cells(1, 1).FormatConditions.add(xlCellValue, xlGreater, "=CP_HIGH_MARGIN_DEFINITION")
    .Font.Color = -16744448
    .StopIfTrue = False
End With

' (3) Make article strategy "D" red
With Cells(1, 1).FormatConditions.add(xlCellValue, xlEqual, "=""D""")
    .Font.Bold = True
    .Font.Color = -16776961
    .StopIfTrue = False
End With

' (4) Make article strategy "A" blue
With Cells(1, 1).FormatConditions.add(xlCellValue, xlEqual, "=""A""")
    .Font.Bold = True
    .Font.Color = -10092544
    .StopIfTrue = False
End With

' (5) Make article strategy "W" green
With Cells(1, 1).FormatConditions.add(xlCellValue, xlEqual, "=""W""")
    .Font.Bold = True
    .Font.Color = -16744448
    .StopIfTrue = False
End With

' (6) Show special cost in bold green font
With Cells(1, 1).FormatConditions.add(xlCellValue, xlNotEqual, "=0")
    .Font.Bold = True
    .Font.Color = -16744448
    .StopIfTrue = False
End With

' (7) Highlight duplicate heading names. There can be none.
With Cells(1, 1).FormatConditions.AddUniqueValues
    .DupeUnique = xlDuplicate
    .Font.Color = -16383844
    .Interior.Color = 13551615
    .StopIfTrue = False
End With

' (8) Make heading rows bold with yellow background
With Cells(1, 1).FormatConditions.add(Type:=xlExpression, Formula1:="=IF($B8=""H"";TRUE;FALSE)")
    .Font.Bold = True
    .Interior.Color = 13434879
    .StopIfTrue = False
End With

' Modify the "Applies To" ranges
Cells.FormatConditions(1).ModifyAppliesToRange Range("O8:P507")
Cells.FormatConditions(2).ModifyAppliesToRange Range("O8:O507")
Cells.FormatConditions(3).ModifyAppliesToRange Range("B8:B507")
Cells.FormatConditions(4).ModifyAppliesToRange Range("B8:B507")
Cells.FormatConditions(5).ModifyAppliesToRange Range("B8:B507")
Cells.FormatConditions(6).ModifyAppliesToRange Range("E8:E507")
Cells.FormatConditions(7).ModifyAppliesToRange Range("A7:AE7")
Cells.FormatConditions(8).ModifyAppliesToRange Range("B8:L507")


ErrHandler:
Application.EnableEvents = False

End Sub

Getting the array length of a 2D array in Java

If you have this array:

String [][] example = {{{"Please!", "Thanks"}, {"Hello!", "Hey", "Hi!"}},
                       {{"Why?", "Where?", "When?", "Who?"}, {"Yes!"}}};

You can do this:

example.length;

= 2

example[0].length;

= 2

example[1].length;

= 2

example[0][1].length;

= 3

example[1][0].length;

= 4

Convert string to date in Swift

Sometimes, converting string to Date in swift can result to return nil so that you should add "!" mark to format.date function!

let dateFormatterUK = DateFormatter()
dateFormatterUK.dateFormat = "dd-MM-yyyy"

let stringDate = "11-03-2018"
let date = dateFormatterUK.date(from: stringDate)!

Setting up a cron job in Windows

  1. Make sure you logged on as an administrator or you have the same access as an administrator.
  2. Start->Control Panel->System and Security->Administrative Tools->Task Scheduler
  3. Action->Create Basic Task->Type a name and Click Next
  4. Follow through the wizard.

PL/SQL, how to escape single quote in a string?

You can use literal quoting:

stmt := q'[insert into MY_TBL (Col) values('ER0002')]';

Documentation for literals can be found here.

Alternatively, you can use two quotes to denote a single quote:

stmt := 'insert into MY_TBL (Col) values(''ER0002'')';

The literal quoting mechanism with the Q syntax is more flexible and readable, IMO.

SQL Server: the maximum number of rows in table

Largest table I've encountered on SQL Server 8 on Windows2003 was 799 million with 5 columns. But whether or not it's good will is to be measured against the SLA and usage case - e.g. load 50-100,000,000 records and see if it still works.

How to change option menu icon in the action bar?

The following lines should be updated in app -> main -> res -> values -> Styles.xml

 <!-- Application theme. -->
<style name="AppTheme" parent="AppBaseTheme">
    <!-- All customizations that are NOT specific to a particular API-level can go here. -->
    <item name="android:actionOverflowButtonStyle">@style/MyActionButtonOverflow</item>
</style>

<!-- Style to replace actionbar overflow icon. set item 'android:actionOverflowButtonStyle' in AppTheme -->
<style name="MyActionButtonOverflow" parent="android:style/Widget.Holo.Light.ActionButton.Overflow">
    <item name="android:src">@drawable/ic_launcher</item>
    <item name="android:background">?android:attr/actionBarItemBackground</item>
    <item name="android:contentDescription">"Lala"</item>
</style>

This is how it can be done. If you want to change the overflow icon in action bar